Discussion file participation PHIL 431 Fall 2008

 

This discussion file is intended as an extension of the collective discussion component of the course. There is never enough time in the class for everyone to say everything worth saying, and not all thoughts occur to one immediately or are most easily expressed in speech. The discussion file lets us deal with all these limitations. To contribute a comment to it, send an email to the instructor, who will then post your comment along with his response. Any relevant new information or new thoughts or arguments about issues in the course are relevant. This includes comments relating to previously posted comments. Each student is expected to contribute at least five comments over the course of the semester. Two of these are due before the midterm. The content of the file is included in the required reading material for the course.

Table of contents (updated weekly)

1. Variations in response to things of aesthetic interest........................1, 12, 15, 80

2. Rules of aesthetic goodness............................................................................2

3. Environmental aesthetics........................................................3, 4, 7 , 25, 35, 39

4. Definition of Art (including categories of art, e.g. architecture).......5, 8, 47-51, 53-55, 57, 61, 66-7, 69, 71, 73, 75, 81, 87-91

5. The ontology of the work of art (including Ingarden)...6, 13, 14, 18, 22, 27-8, 30, 36, 37

6. Radical works of art (their motivation, their value)..........................................9

7. Conservation and restoration practices, aging of art works.....................10, 11

8. The biological (or chemical) basis of aesthetic values .............................16, 33

9. Aesthetic properties, their nature and reality....17, 19, 20, 23, 26, 29, 32, 38, 40, 52, 63-5. (This includes criteria of validity)

10. Kant's theory of beauty..............................................................................21

11. Psychological manipulation in the arts........................................................34

12. Depiction, seeing-in, etc. (including photographs)...................................41, 42

13. Artistic intentions....................................................................................... 43

14. Expression and expressiveness (+Langer)..................................44, 45, 56, 62, 74

15. Essentialism vs non-essentialism about art...................................................46

16. Aesthetic experience..........................................................57-60, 70, 72, 78-80, 82

17. Aesthetic and artistic value (including art and morality)..................76, .77, 83-5.

18. Interpretation.............................................................................68, 92-3

Discussion notes and instructor's replies

1. From John Carter, 9/3/08, re. variations in response to things of aesthetic interest. Thinking a bit more about your example of the two paintings (identical but one judged "beautiful" and one not), I came up with this scenario (although it may cause the example to get outside the realm of what we were initially discussing): Suppose both of the paintings were sold.  One sold for $5,000, and one sold for $12,000 (there was a bidding war during the art auction; the losers who bid on the second copy were much more conservative).  You might look at the $5,000 one and say, "Wow, that's beautiful.  I can see why it sold for so much."  However, you might then look at the other one and go, "What?  There's no way it's worth that much!  It's not even that good."  Both paintings are alike in every respect as paintings -- same composition, same image, same lighting -- but because one sold for an amount you find reasonable and the other for an amount you find unreasonable, it changes your view of them.

JB's response. There are many reasons for people responding differently to paintings or anything else of aesthetic interest. At first the case you describe seemed to me to be one where the aesthetic response is the same -- the difference concerns only the prices paid. But then it occurred to me that you might mean that the person thinks the second painting is not as beautiful as the first ("even" suggests this) because of indignation at the high price paid for it. On the first interpretation the speaker comes off OK, since price is irrelevant to intrinsic value. On the second interpretation the speaker commits an elementary blunder. People do get confused about such things, of course, but that doesn't keep it from being a mistake. Perhaps you presented the case only as an example of human fallibility.

Of course there are special circumstances where no easily detectable differences can make a big difference of artistic value, for instance when one work is an original and the other a supergood copy. But that's because artistic value does not consist solely of aesthetic value.

2. From Caitlin Dietsche, 9/8/08, re. rules of beauty. I was hoping you could clarify our discussion on rules and aesthetic experience. We seemed to agree that there aren't rules for such experiences, like there are moral rules, but at the same time there do seem to be some universal properties. For instance, proportionality and symmetry are aesthetically pleasing. Also, we often use adjectives like "beautiful" or "ugly" for their descriptive quality and others seem to know what we mean and are able to imagine something that way. If there were not universal qualities of beauty or ugliness, we would not be able to do this.

JB's response. You've hit on a huge and highly relevant topic. The question is, what makes a rule? There's no question but that we speak of symmetry and proportionality as being aesthetically good. But can one find sharp-edged rules that will guarantee beauty if strictly followed? Can one even find stateable violations of symmetry or proportionality that will be sure-fire, universal ugliness-makers? Those are the key questions. Doesn't it depend altogether on how a thing of a particular kind is symmetrical or asymmetrical, and what particular kind of thing has this or that set of proportions? If it does, then symmetry and proportionality are not by themselves sufficient to ensure beauty. And in fact they don't seem to be universally necessary. A perfectly symmetrical tree, for instance, might very well be stiff and lifeless looking, and a Janus-headed person would be a monster. The subject is complex, of course. There is a mass of literature claiming great things for the golden section, for example. You will find relevant material on my website under PHIL332 Part I (starting about a third of the way into the document, under applications) in connection with a Platonic theory concerning the sort of beauty that favors symmetry and proportion. Also the articles on Uniformity and Variety on the website contain lots of images where the beauty of the regular polygonal forms involves regularity. Where these properties are needed, deviations from them can be awful, the way sagging or cockeyed Greek or Roman temples would be. But Mondrian's paintings are geometrical but the patterns in them are always asymmetrical and inscrutable in their proportions. Nothing in them is modular, for instance -- resolvable into simple proportionalities. See the examples in the PHIL332 Beauty Notes file, #23. I'll bring some exhibits to class that call into question the efficacy of the golden section for explaining the attractiveness of faces.

And we will meet this general problem of rules frequently during the semester. It's eminently worth close and wide-ranging scrutiny.

3. From Michael Rudman, 9/10/08, re. nature appreciation. In thinking about environmental aesthetics I'd like to summarize what nature appreciation means to me. Imagine staring at a forest of trees from afar. You are limited in what you can appreciate of this forest but your senses are still overwhelmed. As you stare at the lush green leaves on the trees you see them almost collectively dancing in the wind and you hear their rustling. That same breeze then hits across your face and you appreciate the fact that you as well as those trees have just experienced the same wind. Now you approach a particular tree in this forest. You can recognize the particular shape of this tree. The bark acts almost as a protective skin around the tree. The individual leaves take shape over the perfectly random sprouting of branches. The rustling of the leaves become more audible, and if your senses are keen, you can even discern which leaves make which sound. You get even closer to this tree and you see an ant crawling up. You look up and see a bird's nest resting in the branches above and you appreciate the fact this tree is home for animals. Touch the tree. It breathes just like you breathe. It feels just like you feel. It lives just like you live. You look around and realize that every tree is serving the same purpose as this tree. The entire forest is made up of the same kind of tree, but every tree is completely independent. Each tree has its own identity, not so dissimilar from human beings. There is no one right way to appreciate the world. You can see the same thing over and over again from different perspectives and have different but not better senses of appreciation for it. With regard to knowledge about trees, I do not know too much about the origin of trees. For all I know they've been around for as long as life on Earth has existed, so they face the same metaphysical questions humans do. I understand that their function is to provide habitat and food for animals and to generate oxygen through photosynthesis and remove carbon dioxide from the atmosphere. Trees do all the giving and none of the taking. I appreciate trees as well as other plants in nature for that reason. I don't need to know where they come from or why they exist to know that they are beautiful.

JB. Your appreciation is, as you say, perfectly valid even without your having comprehensive knowledge. It sounds like a limited form of immersion appreciation -- though I can't tell from your brief account how extensive or limited it is. Probably you will extend it as you experience more and learn more about the denizens of the forest. I certainly agree that you don't need to appreciate all the beauties of the forest in order to appreciate some of them. But this leaves open the question of what ideal appreciation would be like. A sincere and deep admiration of nature would seem to involve an interest in an ever widening and deepening knowledge of the things you find beautiful in this or that respect. It also seems to imply respect and admiration for those who appreciate beauties that one doesn't yet grasp and perhaps may never grasp. Would you agree?

A companion line of questioning concerns the negative: what do you find unbeautiful in nature?

4. From John Carter, 9/13/08, re. apocalyptic natural beauty . In the lecture notes for last class you asked: "Suppose we knew that all human life was to be destroyed in a future collision and never reestablished. Would we still be justified in aesthetically admiring the natural system?"  I would say "yes, all the more so".  Just as someone might look at a beautiful painting or a beautiful sunset and proclaim "Glorious!", if I were to live to witness such widespread and thorough destruction, I could think of no other description but that: "Glorious".  One could say that destruction is the ultimate form of beauty ("ultimate" in the sense that, after the destruction of everything, there is nothing left to be beautiful).  Is a supernova not beautiful?  It's the death of a star and will wipe out every planet in that solar system, but I think, if we were able to witness it (at least, more clearly than the technology we have available to us), it would be more beautiful than any (comparatively) mundane sunrise or sunset.  You mentioned the mushroom cloud from the atomic bomb... I'd say that every part of the blast -- the cloud, the flash, the shockwave -- was beautiful (the resulting radiation fallout, not so much).  I think that the End of the World will be beautiful.

JB's response. You find beauty in the apocalypse, but that's because you specify that it have beautiful properties: brilliant explosion of light or whatever. Suppose the global biosphere just petered out, increasing cold finally doing in all life-sustaining organic forms. Maybe you could find some beauty even in that -- but it wouldn't likely be overall beauty of the end stages, only selected properties at most, e.g., smooth glacial surfaces or whatever. But that aside, you do affirm the objective beauty of things even without their being favorable to humans, which is one stance to take about natural beauty -- along the lines of G.E. Moore's thought experiment. I wonder whether others in the class agree with you. Dissenters are invited to speak up and give their reasons.

5. From Caitlin Dietsche, 10/7/08, re. the definition of art. In class, we discussed definitions of art. Many of the definitions required that the artist have training in his field or have received some formal education in the art world to be taken seriously. My questions is then, why do artists need to be trained in order for a piece of art to be credible? Why isn't the piece of art itself judged for its intrinsic artistic value? Why must the artist and his background factor into how his art is perceived? From these definitions, it seems art philosophers accuse the viewer of not being able to judge something as art, unless he or she is told that it is art from the start. Why can't we trust our own aesthetic reactions, instead of relying on what we are told about the artist, to make our own conclusions about the qualifications of the piece of art?

JB's response. You raise two questions, one about the art work and another about the viewer's reliability in responding to it. (1) As to the relevance of training, remember that these are conditions on typical art works. So construed training is certainly typical. Typically there is a fair amount of formal training. It shows in the intrinsic properties of the work. In principle there could be an untrained "wunderkind" who amazed everyone by being precocious. And unquestionably there are people who are wonderfully gifted from early years. Work by such persons is not rejected, but invariably their interest in art leads them to acquire training in order to mature artistically. A first-rate artist who never had any training at all, and who didn't study the work of other artists closely, is something that never actually happens.

(2) As to the audience, people learn a lot about art by their immersion in general culture, but they gain terrifically by more specific instruction, whether that instruction be by classes or books or magazine articles or museum commentaries or what can be gathered from museum installations or the fame of artists, etc.. This kind of instruction is hard to avoid and there's no reason to insulate oneself from it, is there? The importance of artists' motivation and the place of their work in the on-going history of art comes from the fact that without some knowledge of this the viewer often doesn't have a clue as to what is worth attending to in the work. Generally this leads to indiscriminate undervaluing of the work, whereas greater knowledge leads to responses that are tied to specific qualities.

I don't mean that clueless responses have zero value. By no means. But they are only a starting point for anyone really interested in art.

6. From Corey Checketts, 10/7/08, re. the ontology of the work of art. In the opening pages of chapter 6, Stecker raises an interesting question where he asks: “So what exactly is the film?” I would like some further clarification on this very question. Suppose that every single version of your favorite film was destroyed, perhaps by a tyrannical government, an Orwellian society similar to 1984, would the art work cease to exist? Stecker seems to be saying that it would continue to exist out there floating in space, that you cannot truly touch the real work of art, that the sense of a work literature, music, or film goes beyond the medium used to express it. But how can anyone new appreciate this work of art if all of them were destroyed? I feel as if this is a crucial part of an object of art, that it has to be accessible. Another question of a similar nature, how is anyone capable of appreciating this work of art? Surely if someone only goes by their memory in order to appreciate the object then there are problems, as one's memory can fade, the work can become less vivid, and etc. so how can that individual continue to appreciate a work of art? Perhaps there is some physical element to a piece music, or perhaps I am off base in my questions?

JB's response. The standard reply to your perplexity is to point to the distinction between the work existing and people being able to appreciate it. This distinction is unquestionably applicable to a painting that is created but then hidden away, lost and never seen again but which continues to exist in its hiding place. The further question is whether it applies to the film that is irretrievably destroyed as you imagined your film to be. Then for anyone to say it still exists requires a theory and one which doesn't depend on the mere possibility of being experienced and appreciated. Many philosophers have claimed that works of art -- all of them -- are essentially "designs" rather than physical particulars, that if in some parallel universe or in ours at a later time a film just like (identical in all its intrinsic properties) yours was made then it would be the very same film and not just one like yours. One more step and theorists (some of them) arrive at the view that the film is an eternal abstract object, like a Platonic Form! The artist discovers it just as geometers discovered the Golden Section. Art works aren't really made although we may be able to gain access to them only by painting or sculpting or shooting film. Wow!

7. From John Kim, 10/7/08, re. environmental aesthetics. In your response of Michael Rudman's comments, you propose the following line of questioning: "What do you find unbeautiful in nature?".  In many occasions, things we find lacking beauty in nature are not the result of lacking in things we associate as beautiful properties.  They may be perfectly symmetrical and proportional, with vivid colors and combinations, but yet provoke an unpleasant aesthetic response.  Spiders, snakes, and bugs, to many people, will provoke a negative response.  For that, I feel there may be reasons greater than aesthetic value.  Unfamiliarity also seems to play a large role in beauty judgment.  People who work with these creatures many times have an aesthetic appreciation for them that is counter to intuition.  Tied into this is that because aesthetic response is more so felt than explicitly delineated, our aesthetic response may be irrepressibly tied into some sort of evolutionary response.  People who are around these creatures often can develop a new sense of aesthetic and overall intuition towards them, and thereby eliminate negative responses to them.  This seems to implicate that aesthetic response is able to be influenced a great deal, subject to opinions.  I can't seem to fully reconcile this subjectivity with the supposed objectivity of aesthetics.  I agree that such objectivity exists, but when it is seemingly thrown out the window when overridden by other natural responses, I have difficulty judging the line between subjectivity and objectivity in aesthetics.

JB's response. Unquestionably most people find most insects unlovely, and when the insects are magnified enough for their intricate features to be clearly seen, the aversion to them increases. That seems partly because they are thought of as dangerous or dirty or bothersome. Only a few especially pretty insects are regarded comfortably by the majority of folks. However partly the aversion is to their body structure and especially their face, as well as their typically hard outer surface. They aren't regarded as good looking. We can say with some confidence which quadrupeds are good looking (foxes, Arabians) and which are not (hyenas, donkeys) and what would improve the appearance of the latter. But we have a real problem imagining what a good looking cockroach or bee or mosquito would be like. I think these are the major factors that count heavily against aesthetic acceptance of insects. Ladybugs are accepted because their form is neat and nicely colored. Butterflies give delight because of their wings and whimsical flight. Neither is believed dangerous, dirty or bothersome.

Entomologists are able to rise above considerations of self-defense and take the insects on their own terms. That seems only fair, since we do that with animals we accept as beautiful, such as leopards. Do entomologists find beauty of appearance in the insects cited above? I find that doubtful. I think they select other properties for admiration, mainly functional properties. The flight of insects is stupendous, as is the water-striding of water-striders. Daddy longlegs are terrifically strong in the leg. Ants and termites are wonderfully well organized. But these qualities are not generally thought of as beauties by outsiders. Isn't that the problem?

The question is, then, do these functional properties deserve to be admired aesthetically? If they do, then they plus such beautiful appearances that insects arguably do have may provide a solid basis for insect beauty. How this beauty compares with the beauty of the species we favor, our favorite birds and beasts -- and of course our own kind -- is a further question, isn't it? The main thing, I think, is not to remain mired in too narrow a view of beauty. Rise above that.

8. From Morris Panitz, 10/13/08, re. the work of art. In one of the responses on the discussion board, you wrote, "Many philosophers have claimed that works of art -- all of them -- are essentially "designs" rather than physical particulars, that if in some parallel universe or in ours at a later time a film just like (identical in all its intrinsic properties) yours was made then it would be the very same film and not just one like yours." This comment seems to contradict Stecker's assertion in chapter 6 that, "a work has crucial artistic properties that derive not from their structure but from their origin." It seems to me that point is extremely relevant to the art world because fraud, originality, and authenticity are of such great importance. Don't the conditions surrounding the creation of the artwork (historical, personal, cultural) profoundly alter the essence of the artwork even if the other properties are identical? This seems to closely mirror the discussion about knowledge and nature appreciation, in which one conclusion stated that our appreciative experience varies with degrees of knowledge.

JB's response. The issue is a tricky one, which is why there is so much controversy about it. I favor Stecker's position but the opposition also has its thoughtful and well informed advocates. One sticky problem is what counts as an intrinsic property of the work and what is only a contextual circumstance. Is meaning intrinsic? If it is, then whatever social context is necessary for the meaning of the work has to be replicated in the alternate universe for the intrinsic properties to be replicated. That will much reduce the resistance to saying the two are the same work, won't it? The "design" will be more than just the physical features. But the experiment will seem much farther from reality if the alternate universe has to have the same meaning-conferring social context.

Another thing defenders of the design idea stress is the popular overemphasis on who created, or who owns, the work. They view that as a celebrity fixation. They may push this point too far but the danger of artist-worship is real enough. When a trifling scribble by Picasso brings a huge price things are a bit out of joint. Similarly when a "minor" artist produces a work almost as good as a work of a major artist, typically it is valued at a tenth as much as the other. The design theory tries to purge our thinking of these distortions.

9. From Nathaniel Snyder, 10/13/08, re. radical works of art. Concerning Duchamp's "Fountain," and your discussion of the aversion to bodily function in aesthetic pleasure. Did Duchamp have that in mind when he "created" this piece? Is it simply coincidence that he not only created a piece to counter traditional ideals of art, but also one to defy the aesthetic pleasure of art?

JB's response. Certainly "Fountain" was more offensive because of its association with urination. And Duchamp certainly was aware of that. On the other hand, the other found objects he exhibited during the same period had no connection with excretion or other "unmentionables." The snow shovel, the bottle rack, etc. were comparatively neutral on that score. I would say that the commonplace utilitarian character of "Fountain" was at least as important, but I agree that the more specific scatalogical reference spiced it up.

10. From Nathaniel Snyder, 10/13/08, re. reconstruction of damaged works. This question concerns damage and subsequent reconstruction especially of statuary. You showed the class a Venus statue. While in Italy, I saw this piece, but I also remember a series of Greek statuary that suffered even more extensive damage in the same gallery. The discus thrower, which had suffered attempted reconstructions, some which ended up portraying the statue figure awkwardly holding a shield and a sword or some such scene. I would think that this erroneous reconstruction should give some pause to even attempting to rebuild damaged art, lest the original intent be lost. Better to let things be as they lie? Thoughts?

JB's response. There are two questions here. First, what should one do when one knows quite well how the work was originally? Second, should one reconstruct speculatively when the evidence is less definitive? Preservationists (or conservationists) favor not restoring at all, just preventing further deterioration. Restorationists favor intervening, filling the cracks and crevices and reassembling the pieces in as plausible a way as possible. Now of course there are gradations of both general approaches. Preservationists can favor reassembly when the parts are available, the breaks are clean and there is no uncertainty how it was originally. They may also favor cleaning where the statue is stained or defaced with graffiti. Restorationists can limit their interventions to those that are strongly backed by evidence. In the other direction extreme preservationists may not want us to venerate the signs of wear and tear or defacement or anything else.

The mainstream museum practice is never to make speculative restorations but to reassemble where it can be done with relative certainty, to keep ancient wear and tear but to repair works recently damaged (e.g., Michelangelo's Vatican Pieta). That seems to me a reasonable practice. Speculative restorations can be done via copies for educational purposes, so there's no reason for involving the originals in that. The success of such efforts will vary greatly, but in many cases the enterprise is perfectly valid. An interest in the work is in part an interest in how it was originally. There's no other way of satisfying that interest except through a good effort at replication. The art world doesn't actually do much of that now for a variety of reasons, but it would be a good thing if it could do more of it (without sacrificing other higher priority tasks).

11. From Brian Morris, 10/14/08, re. damaged works of art. Last class we started to talk about how damage affects the aesthetic value of an object. I was wondering if it is possible for the damage to add some aesthetic value to the object. For example the Statue of Liberty has gained a greenish color over the years because the copper has been exposed to water. However, now it seems that everyone associates a green color with the statue even though it may not have been that color originally. Thus again my question is can the addition of the color add to the aesthetic value?

JB's response. Ageing causes all sorts of changes and works acquire different properties thereby. How constructive they are, from an aesthetic point of view, varies a lot. I don't think there is any general rule, since there are so many different considerations. Oriental carpets are often more prized when their bright colors have faded into soft and subtle ones. But many Venetian paintings are the worse for some of their greens turning brown. Copper turning green is a well known phenomenon, so no artist could have expected anything else. If the choice was a good one at the time of creation that process must have been considered good. And indeed that subdued green goes pretty well with the harbor scene. You could try to imagine other colors and judge comparatively. Is there any other color that is clearly superior. Would unoxidized copper be better? I doubt it. (I don't know its history, but I'll look it up.)

12. From Corey Checketts, 10/14/08, re. It seems as if our appreciation of something; our conception of the aesthetic is dependent on our ability to observe (or our aesthetic experience) any given object. Take for example certain Inuit tribes/cultures; in their native language they have far more words for the description of various types of snow, snow's properties, patterns, and etc. than most of us can even imagine, and it seems plausible that they appreciate snow and its various aesthetic properties, and that they appreciate these in a much more acute manner than say you or I. Further, I don't think that most would dismiss their observations as an aesthetic experience, but the nature of my question is as follows: what gives way for their superior descriptive capabilities? Why can they describe and appreciate snow in a way far beyond the average human? Is it because they are surrounded by snow most of the year, due to their geographical location? Does location play some role in our appreciation of certain things? It seems as if our surroundings does imply and affect what we consider beautiful, at least to some extent. A society of sheep herders will have a greater understanding and appreciation of sheep than you or I; they are more capable of “seeing” the aesthetic characteristics of a sheep in this example. If our environment does help determine our aesthetic appreciation in the sense of what and how we appreciate a given object then, what can be said of a culture surrounded by what most would consider ugly? Could a culture appreciate something aesthetically which is ugly? An example continually given in class was insects; under a microscope we can find something beautiful there in our observation of an insect: their proportionality, symmetry, exotic color, and etc., but for many insects are ugly vile creatures. So are these creatures ugly? Or is there more to something more to our understanding of aesthetic properties? Can we apply a different “lens” and then see the ugly as beautiful? Could a rather grotesque creature be appreciated for aesthetically pleasing properties which others cannot adequately see or comprehend based on one's relevant environment?

JB's response. Lots of questions here! First as to different sensitivities to sensory qualities, these can certainly affect what we appreciate. If you don't notice different types of snow or gradations of white you won't be in a position to appreciate the differences. On the other hand more names doesn't necessarily mean greater capacity to detect differences. And sharper discrimination won't automatically transfer into greater or subtler appreciation. I said long ago that different aesthetic tastes sometimes result from different expertise at recognizing patterns -- different societies develop different specializations. I think that covers your cases. It's not that people differ in their innate capacities but only that they are trained to notice different things.Second, what people like best aesthetically is more complicated. I really don't know what would result from people being immersed in a world of insects (especially big ones!), but if the people got to the point of feeling at home there it is reasonable to suppose that they would appreciate lots of insect properties, and hence transcend the vulgar revulsion that is so common among us. A lot depends on how comfortable they are among the insects. But assuming they did feel at home, it's unikely they would relish insect faces -- mandibles, proboscises, antennae, eyes and all. There is reason to think that they would appreciate the features we can also appreciate (with a little effort). So I would not anticipate any relish of the truly ugly.

13. From John Kim, 10/16/08, re. the constructivist paradigm of the work of art. When Stecker responds to the constructivists' paradigm, he separates the object being interpreted from the interpretation.  He states that "supposing that interpretations construct subsequent objects is a prime example of confusing or conflating the interpretation and object of interpretation."  By this, he suggests that the initial object was nonintentional, and that there is no need to "hypothesize" the subsequent object, that we need only to "select features" from the initial object and interpret them to accomplish understanding.  I don't quite follow Stecker here.  Isn't taking the "feature-selecting interpretation" basically equal to positing a subsequent intentional object?  What does Stecker exactly mean when he states that there is no need to postulate a subsequent object, if "feature selecting interpretation" does not fall into the definition of postulating a subsequent object?

JB's response. A highly relevant question. It's true that a painting, say, which has an intentional property (a meaning of some sort, for instance) is from the outset an object that has more than physical properties: it has meanings from the beginning. So there are grounds for regarding it as more than merely physical, or as a hybrid object. The question for constructivists is whether a new interpretation, one not based on the original context of production, creates a new object. Indeed in postulating that new property an interpreter can be said to postulate a new object, but that postulation doesn't thereby change the original work of art. It just misinterprets it. That's Stecker's essential point.

This is a matter I didn't give enough stress to in the last class, but it's stated in the reserve reading for Ingarden. A painting or sculpture with various intentional properties (meanings of various sorts) is still properly classed as a unique physical object because any other painting will be a different one even if (miraculously!) all the physical and intentional properties of that other are the same. The physical uniqueness of the original is sufficient to guarantee that -- at least it is for all of us who reject the "design" theory of works like autograph paintings, sculptures and the like. Works of music and literature have a more complicated story but the contextualist account will make the same criticism of constructivism -- interpretations not founded upon the original context of production are invalid. They tacitly postulate a new work that has no historical reality

14. From Peter Frechette, 10/18/08, re. creation vs. discovery of the work of art. We have talked about the discovery of a design or of a possibility as being part of what art entails. Just as Newton and Einstein did not truly invent laws of physics, do people really invent art, or merely discover it?
  All children discover that three dots and a curved line can represent a face, but three dots and a curved line also occur in nature, (which is why some rock and cloud formations tend to look like faces), so the children certainly do not invent it. Ancient Egyptians and South Americans discovered the pyramid at around the same time, they did not invent this arrangement of stone blocks. In fact, I think all arrangement of materials are discovered, not invented. However, I also believe that art takes a form of invention or creation. So, I think that while a painter might discover an arrangement of colors, or a writer, words, which was already possible, he creates the feeling behind the colors, which is why an artist can turn almost any scene into a picture frought with meaning and feeling. It is not the beauty we discover, but the meaning we give it which makes it art.

JB's response. I agree with much of what you say. It is significant that that the most relevant "discovery" in art is not of a thing or process but of a fact -- which is expressed by a propositional clause (in logic a "propositional function"). We discover that this or that configuration of lines or sounds has certain aesthetic or other artistically relevant properties, beauty, emotional expressiveness, etc. This is essentially hypothetical: if this pattern is embodied, it will be beautiful, etc. Creation (or invention) in art, on the other hand, is best thought of in terms of things: real objects or real processes. The artist creates the work when she dreams up a melody or works out a visual design by drawing, painting or whatever. The creating is the musical planning, the humming to oneself or working out the piece on an instrument, writing it down or just remembering it -- or the equivalent in visual media. In principle it's possible for it to be all in one's mind, but in practice all sorts of physical aids are used. Thus the work is the historically unique outcome of physical and mental activity -- the physical work plus its meanings within the artistic context of the time.

Of course every work coincidentally calls attention to a general design that is an eternal possibility, one that can be replicated endlessly -- but the work itself is not that general design but the historically individual exemplification (or in music and literature a class of exemplifications tied to a historical origin).

In the empirical sciences discovery that also applies: that X causes Y or that X is composed of Y, etc. But here we have an already embodied relationship, one that actually obtained in the world before its discovery. It wasn't just a possibility. What is invented is the manner of formulating the relationship (plus the instruments and procedures for measuring and otherwise dealing with it).

15. From Daniel Eliezer, 10/18/08, re. "rational" subjectivism. I'm having trouble accepting a position Stecker calls “complex” or “sophisticated” subjectivism. Stecker thinks that this is the only antirealist (or anti-aesthetic properties) position worth considering because it offers a normative element. This means that while two contrary impressions of a given object may both be valuable, as subjectivism implies, one impression may be more valuable than the other if we consider possible defects in impression. Stecker gives an example of an observer's impression of an object being defective, and therefore less valid, if he or she is in love with the artist. And this is comparable to the ideal observer theory since we now have guidelines for judging just how valuable an impression really is. But here's the problem I'm having. How is this validity measured? How is it decided whether the observer's evaluation is blameless or defective? Are there objective guidelines for determining this? If there is an objective means of measuring validity, what is it? Subjectivism denies aesthetic properties because it argues that even ideal observers may have different impressions of the object of interest (meaning that there are no unique dispositions in objects; and thus, no aesthetic properties). The same argument can be made to attack the notion of an objective guideline for measuring the validity of an observer's evaluation. Suppose an observer is an American historian with a positive impression of an artwork depicting an event in American history. Is this impression more or less valid than an observer who isn't an American historian? There appears to be room for disagreement. Judge 1 may claim that the observer's love of history and familiarity with the event depicted are defects that bias him, and his evaluation is therefore less valid than a non-American historian without these defects. But Judge 2 may argue that the historian's impression is more valid because only observers with a background and familiarity of the historical event can most suitably evaluate it, making the non-American historian's impression defective and less valid. So which of the two guidelines do we follow for measuring the validity of the observers' impressions: Judge 1 or Judge 2? It seems only reasonable to conclude that both judges, and both guidelines, are equally valid or incommensurable. If we challenge the validity of one judge's judgment over the other, we are led into a vicious regress (we can keep challenging the challengers forever). This means that there is no single objective guideline for measuring validity. And this should reduce complex subjectivism into a non-normative simple subjectivism since we are unable to determine whether or not the observer's impression is defective.

JB's response. Like the previous item, this one is difficult. I can certainly appreciate that a subjectivist could be more or less "sophisticated" or as I prefer, "rational." That would mean preferring pleasure from error-free perceptions to those that are based on errors, and pleasures that are sustainable to those that are fleeting or that leave a bitter aftertaste, and pleasures that fit into a coherent life rather than ones that lead to incoherence, inner conflict and the like (pleasures from drugs or alcohol are a good example of this). Even from an entirely egocentric point of view, these preferences make sense. In ethics one speaks of "enlightened self-interest." That's parallel to rational subjectivism.

A further note abour skepticism about aesthetic properties is also in order here. Skeptics may reasonably doubt the objectivity (or better, the intersubjectivity) of lots of aesthetic property claims, but there are also lots of claims that are unreasonable to doubt. The temperature effect of the basic hot-cool colors is so powerful and so universal that it is highly implausible to doubt its standing as a human universal. That is, all humans who are cognitively competent with regard to color and temperature properties are capable of experiencing paradigmatically hot/cool colors as hot/cool, and they are not capable of experiencing them in the reverse way. So it is for many descriptive aesthetic properties. So also for paradigmatic cases of beauty and ugliness, as those exhibits I showed at the beginning of the course make evident. The only reasonable controversy concerns where to draw the line between the universal and the less-than-universal capability.

16. From Matt Bakalar, 10/19/08, re. evolutionary advantage of aesthetic sensibility. We talked briefly about the argument from evolutionary advantage earlier on in the course. The argument addresses the question of why we value aesthetic experience. Throughout the course we've been presented with different experiences that people consider aesthetic. We've been asked to identify common the common properties of these experiences, differentiate them from lower level pleasing sensual experiences (is enjoying a good meal an aesthetic experience?), and to locate the aesthetic object that is the target of our appreciation. In answering all of these questions, we seem to rely on our own aesthetic compass to make value judgements, searching for definitions that align with most of our intuitions about what an aesthetic experience should be. But we haven't asked what seems to me a much more interesting question, why do we find some things beautiful? There must be a biological basis for our experience of beauty. Do our experiences of beauty provide us with some direct evolutionary advantage, or are such experiences side effects of some other feature of our intelligence, a wonderful accident? Even if there are answers other than evolutionary advantage, isn't the question worth asking? Shouldn't this preclude our arguments about what particulars are aesthetic?

JB's response. I can certainly agree that the question of evolutionary advantage of our having aesthetic sensibilities is a valid and interesting one. But I can't see why that should upstage questions about what is aesthetically valuable. In fact the biological question seems to presuppose the answer to the more central questions -- central because it provides the target of explanation. We have to begin by identifying shared aesthetic values and then ask, what evolutionary advantage does our valuing these things have. It isn't conceivable that we go the other way and come up with aesthetic values radically different from the ones we recognize. The biological gambit, as it might be called, seems to me an evasion of the more essential job of aesthetic analysis, an attempt to somehow establish certain values on an essentially non-aesthetic, scientific basis. To my mind the only rational way to proceed is to see how much convergence serious aesthetic observers can achieve under cognitively optimal conditions. That involves using our aesthetic sensibilities to the max, not making an end run around them.

That said, it does seem plausible that shared aesthetic enjoyment is a considerable factor in making life worth living and thereby motivating us to overcome challenges to survival.

17. From Tracy Yau, 10/19/08, re. beauty as a positive value. As I was reviewing my notes on aesthetic properties I thought Stecker makes an interesting point that expressive, behavioral, and most of the remaining second order perceptual and representatational properties don't inherently have positive or negative value. Stecker gives the example of a piece of art being graceful. This property is usually seen as something positive but when put in context with a painting that is meant to display the brutality of war, the property of gracefulness detracts from the overall value of the painting. However, I was thinking, couldn't we say the same thing about the painting being beautiful? If the painting were to be proclaimed as something beautiful, wouldn't that also lessen the value of the painting when the author's emphasis was meant to be something brutal and perhaps even ugly? Or is it because the property of beauty is so broad and general (and seen as an evalution of the painting) that it does not fall under the same problem as the descriptive properties?

JB's response. That's an interesting question. "Beauty" is a term of highly uncertain scope. Often it is used for what is easy to love, the sweet, lovely, graceful, harmonious, comfortable, reassuring, and in general the immediately attractive. But there is a wider, more philosophic sense in which it is based on what Guy Sircello calls the adverbial usage, namely "beautifully." Here whatever has an aesthetic property beautifully counts as a thing which is beautiful in respect of that property. Hence if a harsh or even excruciatingly painful work of art is highly and non-defectively expressive, it can count as beautifully expressive. To be beautiful all things considered, or overall, or as Sircello says simpliciter, is another matter. So a piece of music or a drama that is hard to take will not necessarily be beautiful simpliciter even if it is expressively beautiful. I find this general line of thought persuasive.We don't have any other term as good as "beautiful" for the hard-to-take but worthy aesthetic values, so I favor using the term -- with discretion. The last implies saying in what respect it is beautiful and how it stands overall when put up against other things of its kind. Just saying it's beautiful says far too little to convey the essential points.

18. From Tracy Yau, 10/19/08, re. the work of art. When we were discussing Levinson's indicated structures, you stated that if we were to digitize and speed up a piece of Mozart, that it is not really performing Mozart. Instead, it would be considered a parody. However, I was thinking about the Trans-siberian orchestra. They digitize a bunch of classical music but I do not think they have the intent at poking fun at these classical pieces. Instead, I feel that it is just taking the classical piece and changing the genre of the music to another and making it their own. Couldn't we say the same about a piece played by the National Symphony Orchestra that emphasizes certain parts of the piece to make it their own? Could we still not call both performances one of Mozart?

JB's response. I assumed that the speeding up was radical, way beyond the limits of acceptable performance. Say, at least twice the normal tempo. This makes a huge difference, whether the effect is parody or something else -- and notice, it's got to have some recognizable motivation or else it wouldn't be a performance of anything. There's another wrinkle in this subject: there are musical works based on other works, for instance variations, e.g. Brahms's variations on a theme of Haydn. Performing that isn't performing the Haydn but the Brahms. In general when the deviation from normality is big enough there's often another category to put it in. From your closing remark, though, I'm unsure of the range of cases you have in mind. Certainly an orchestra can vary the tempo or other dynamics, or the orchestration, etc. of a piece without crossing the line to another work. Idiosyncratic performances (interpretations, they are called) can still be performances of the work, but only within limits that are pretty well understood by the musical world.

Incidentally, digitizing and playing on a synthesizer is already, strictly speaking, a reproduction rather than a performance, so my example in class was a bit off. But it still illustrates the larger problem since it's a way of conveying a musical work, or of conveying a near relative of it, as the case may be.

19. From Mike Gabaly, 10/20/08, re. aesthetic property criteria of validity. The closing paragraph of chapter 4 discusses labeling aesthetic properties if they , "...have or not have a steady disposition to produce the relevant reactions in ideally situated observers?" Does this mean that an aesthetic property can only exist if it can continually provide the ideal observer(s) with the same response?

JB's response. Strictly speaking the criteria only require repeatability under optimal conditions, which means optimality of the viewer's position and condition. Deficiency in either aspect disqualifies the test. Now obviously it won't be much good if we set the optimality threshold too high, since then we'd never achieve it. But it also mustn't be too low, or meeting it wouldn't filter out defective responses. Even highly qualified observers can make mistakes, fail to discriminate different sensory properties in the base of an aesthetic property and fail to detect the cross-categorial resemblances and where relevant feel the aesthetic satisfaction that is appropriate. The only possible outcome will take the form of statistical frequencies that are high enough under realizable test conditions to justify belief in the aesthetic property being a reality. As in cognitive enterprises generally belief is based on the best explanation of the data. Some aesthetic properties (strictly, aesthetic property distinctions) will be universally upheld and therefore solid. Others will be less so, down to the dubiously objective. At the lowest level there are merely subjective impressions -- phobias and idiosyncratic associations are examples.

20. From Mike Gabaly, 10/20/08, re. aesthetic property criteria again. When trying to ensure a given aesthetic property exists even if two observers have differing views, is it safe to say as long as they can reach a general agreement the property exists?

JB's response. The answer here is the same as to the preceding. Optimal conditions will produce agreement in enough cases to support the claim that the property exists. The conditions should include opportunity for the parties trading explanations of their responses, that is, both of what is experienced and how. It isn't simply a matter of "I agree/disagree" but an extended process of discovery. It also presupposes a history of shared responses of less contestable examples.

21. From Erin Whiting, 10/20/08, re. Kant's theory of beauty. In chapter 3 Stecker lays out Kant's four criteria of a positive aesthetic experience, one of which is subjectivity, "that is, they are based on a felt response of pleasure rather than the application of a rule or concept." I don't see how subjectivity, which is internal and can vary preferences based on an individual's psychological background, can coexist as a standard along with universality, which is "the claim that others ought to judge or respond similarly." I can understand Kant not laying out a specific set of rules, instead saying that rational people judging from reason would all come to the same conclusion, but that to me is not subjective; it's just saying that judgments of the aesthetic, like judgments of morality, are based on common sense. How can personal pleasure be a factor in reaching a universal consensus?

JB's response. Kant realized his idea of (free) beauty sounds paradoxical in just the way you suggest. But his idea was that certain pleasure-responses are an essential part of cognition and thus universal among all cognitively compentent experiencers. In particular a pleasurable harmony of the cognitive powers (imagination and conceptual understanding) at a pre-conceptual level was essential. That harmony occurs when we contemplate well-composed patterns even when the patterns aren't pictures of anything or illustrations of concepts (as in geometry). The goodness of the organization is that it (a) stimulates the imagination and sends the understanding in search of concepts and (b) pleases the two even when no concept is found. Think of a melody in music. It may be charming without representing anything and without illustrating any concepts. Hearing the first part we project good continuations (that's our imagination) and hope for a good completion that seems well ordered even though there's no actual concept that we can cite explaining its goodness. Similarly, purely formal compositional goodness of visual materials brings the two faculties into a pleasurable harmonious state. This state is a precondition for forming concepts of objects in a conceptually coherent spatial setting and events in a conceptually well-ordered temporal setting. Free beauty thus forms part of the basis of concept-formation. We can reasonably expect everyone to share in this pleasure. An important additional point is that the pleasure is so familiar that it is hardly noticeable. It's miles removed from Bell's ecstasy.

As Stecker says, this will not cover the field of beauty. Far from it. It's only one kind. The more interesting category is "dependent" beauty, which brings in other elements. As to pleasure being subjective, however, please note that pleasure that is universally shared under optimal conditions (ideal observer conditions) has an entitlement to being considered intersubjective -- or for practical purposes "objective." I personally see no way to explain beauty without bringing in aesthetic pleasure of this sort.

22. From Chris Desimone, 10/20/08, re. Ingarden's arguments. I was wondering the difference (if there is) between the meanings of "objectivities" in Ingarden's 1st argument as opposed to "universals" in argument 2''. I'm trying to distinguish between the two because I find the second argument appealing but cannot figure out why he uses two different words to express the same idea. If I am wrong about their similar meaning that would be great to know also because I would not want to be mistaken about aesthetic jargon Ingarden uses. If you could help clarify this issue it would be greatly appreciated.

JB's response. By "objectivities" Ingarden means anything conceived as a object in the broad sense: the object of thought or imagination. It could be a fiction or even an impossible (because contradictory) thing like a round square. I take the argument 2' from footnote 3 on page 7 of the reading where Ingarden points out that in reading a literary work we hardly notice the typographical peculiarities of the text but just take in the typical verbal forms. These typical forms are universals rather than particulars. In fact the whole sequence of words constituting a text is a complex universal. For this reason Ingarden doesn't take the work to be identical with the text. The work is a more complicated "objectivity" constructed by the aesthetic experience of viewers in the manner he tries to explain in these pages.

23. From Brian Morris, 10/20/08, re. subjectivist and expressivist theories of aesthetic properties. I'm not sure if I understand the difference between Stecker's subjectivist and expressive views of aesthetic objects. It seems that they both mean the same thing. Could we not just say that we are expressing our subjective views about some aesthetic object? Could you please clarify the difference between the subjective view and the expressive view?

JB's response. The difference is this: a subjectivist thinks that an aesthetic property claim is, or should be, a statement about the person's personal response. Essentially it says I find this music sad or this building pompous or whatever. There is no implied claim that everyone else should agree. They may differ without fault. The expressivist goes one step further, saying that it's not a statement at all but only an emotional expression. It's easiest to illustrate this for beauty. Saying something is beautiful is just a verbal way of sighing appreciatively -- ooooh! A technical term for this interpretation is "non-cognitivist." Needless to say, I strongly disagree with this description of any serious aesthetic property utterance or belief. Sure we can just emote toward a beautiful object, but that's not what we do when we seriously judge it to be beautiful. That is, I take a "cognitivist" stance toward serious aesthetic property claims -- I take them to be statements and therefore true or false (whereas emotional expressions are neither).

24. From Kesun Lee, 10/20/08, re. aesthetic experience topic on the test. I had a question about the exam for PHIL431, on Tuesday October 21st. On number 4 of the 4 possible questions listed, it says to critically discuss the main issues dividing theorists about the definition of aesthetic experiences. Do you have a certain number of theorists to adress in mind? Or should we just pick a few (2-4) to discuss?

JB's response. Yes, I have in mind especially (a) the high-end experience folks -- Ingarden, Bell; (b) the mid-level folk -- Levinson, esp. (c) Stecker's minimal theory; and if there's time (d) the ideas I set forth. This selection is implied by the review lecture outline (Lecture 14). The outline also indicates some of the relevant points to include.

25. From Keegan McClure, 10/20/08, re. environmental aesthetics. Stecker, on page 24, claims, "That knowledge can both enhance and alter the appreciation of nature for individuals does not imply that it must do so uniformly across all individuals".  In fact, he says he believes it does not "because people are differently disposed to be responsive to the countless aspects of nature capable of aesthetic appreciation." This lends me to believe that aesthetic experience is in "the eye of the beholder", but you so ademently denied that possibility on the first day of class. How do you reconcile the fact that knowledge affects individual preceptions of what is beautiful differently with your claim?

JB's response. There's not really a conflict here. Knowledge does not necessarily enhance appreciation, but under optimal conditions relevant knowledge will enable a well-motivated person to recognize aesthetic values otherwise invisible to her. Stecker speaks of what actually happens in ordinary circumstances. My claim concerns what judgments and what appreciative experiences result under optimal conditions.

But there is more to the huge topic of appreciation of nature. Nature is so sprawling and many-leveled a field that one's cognitive system can be overloaded and one's aesthetic sensibility confused. Not all knowledge of natural systems can be put to aesthetic use by a single person. We have to specialize and leave it to others to cultivate other specializations within the realm of environmental aesthetics. An example is the enthusiasms of entomologists, which are a closed book to most people who specialize in other aesthetic realms. We can visit other specialists. Entomologists can visit mountain climbers, say: look over their shoulders and appreciate much of what they appreciate but not all. Some of the fine points in their perceptions will be inaccessible to outsiders, but each group is right to trust the other group's collective judgments. Only if we enter into another's kind of intensive experience of such a specialty will we be able to deal with that person as an equal. We have the capability but we don't have the time! And when we try to embrace all of nature we necessarily fail to have more than a rather foggy, generalized appreciation. It's too vast.

26. From Keegan McClure, 10/20/08, re. aesthetic properties. [Keegan's question was evidently a general one about the distinction among different sorts of aesthetic properties. Somehow I failed to post it.]

JB's response. First note the basic distinction between value-loaded and purely descriptive aesthetic properties. Beauty is a prime example of the former and sadness in music or swiftness of a line are examples of the latter. Many aesthetic properties seem to combine both valuational and descriptive content. Other important points are set forth in Lecture 14 outline. Look over what is said there and formulate more specific questions if any occur to you. The bulk of any test essay on this topic should concern descriptive aesthetic properties. Check out Lectures 7 and 8 and, of course, the relevant material in Stecker

27. From Thomas Wilson, 10/20/08, re. Ingarden's idea of the work of art. During my preparation for the exam, my understanding of Ingarden's essay reached an impasse. According to Ingarden, literary works of art are not physical, psychophysical, nor psychological. That is (from what I understand), literary works of art are not physical texts because texts are merely tools that give the reader access to the work of art, nor are they mental images/conceptualizations because mental images/conceptualizations only refer to a literary work of art. Does this mean that the true “literary work of art” lies somewhere between the physical text and the reader's interpretation of it? In other words, does this mean that the text gives us access to the “literary work of art,” and the mental images are simply our underdeveloped interpretations of it? It seems that Ingarden tells us what the literary work of art is not, but gives little explanation for what it is.

JB's response. Ingarden calls the work of art a constructed "objectivity," which I take to be an "object" of complex mental activity: perception, imagination and thought. It isn't a psychological process or state because it's the thing the states are about -- it's their object, whereas they themselves are processes individual to the person. The work can be shared by any number of persons. In the process of bringing such an object ("objectivity") to full concreteness. He does say quite a bit about what properties the Venus of Milo has. She's a goddess, has no cracks or chips, is animate not marble, is serene, compassionate, etc., and harmoniously embodies a complex overall quality that he doesn't name but says we can recognize for ourselves if we go through the process he describes. Literary works would be basically similar -- the narrative or stream of thought, the stylistic and sonic qualities, the descriptions, figures of speech, the themes are all are part of it and it too has an overall quality -- think of the tone or feel of Henry James or Shakespeare. All this goes into our conception of a novel or play. It's an intentional object, not a real one.

28. From Peter Frechette, 10/20/08, re. the work of art. In class we talked about culturally emergent objects as art. Does this include works of art that emerge from a culture at a point when that culture is going through radical changes, before these changes are really part of that culture? It seems that these objects emerge from a situation, instead of a culture. For example, works that emerge during times of great upheaval, whether in celebration or in mourning, literature and graphic art from the early renaissance or from the Holocaust. In these situations, it seems that the human "situation" is changing faster than the culture involved, especially in the Holocaust, so can a work be said to emerge from a culture when the culture is in such a great flux? From what then, do these works emerge? I believe that some of what are considered culturally emergent objects really emerge from a combination of events, events which a culture is still trying to assimilate.

JB's response. "Culturally emergent" doesn't restrict the creativity of the work. As you say works can appear before the culture is ready for them. But that just means that the more creative spirits can outpace the mainstream, or even the relatively advanced movements within a culture. These avant-avant-garde artists are still deeply dependent on what is going on around them. Try to imagine Picasso suddenly appearing alongside of Rembrandt. It's quite unthinkable. The ground for Picasso was not then prepared. What this means is that the process of emergence can take a number of forms, but it can't be utterly inexplicable. There are always deep linkages. Artists learn from each other, even the wild eccentrics do. And there is always fairly quick uptake by the culture, even if that may be limited to a small circle for a while.

29. From Thomas Wilson, 10/20/08, re. defective aesthetic properties. I am confused about the idea of the 'non-defective' aesthetic property. In your remarks about the exam responses from last semester, you mention that this suggests either positive or negative aesthetic properties. Does this refer to properties that are not degree-based (a work of art can be either three-dimensional or not, without any room for variation)? Or, does it refer to properties that clearly suggest a pleasing or non-pleasing attribute (beauty/repulsiveness are non-defective, while brightness/darkness are not)? Or am I wrong on both counts?

JB's response. Non-defectiveness is part of the criterion of beauty (or other aesthetic value). If a property is a property of defect, such as sickness, stupidity or malice, then it cannot be a beautiful property, even if it satisfies all the other criteria (is a property of degree and is intense). If it's non-defective and has those other qualifications, then it is a beautiful property. That's my suggested criterion anyhow, following the lead of Guy Sircello. The deeper explanation is that such properties are non-defectively pleasant or satisfying under optimal conditions. To be pleased by sickness or death or malice, etc., is to be in a defective state: one's pleasure is defective -- perverse or in the extreme, deranged. The standard for defectiveness is in itself non-aesthetic: medical, psychological, moral, athletic, intellectual, or whatever, so invoking non-defectiveness doesn't make the account of aesthetic value circular.

30. From Alex Culver, 10/20/08, re. contextualism as a theory of the work of art. I know this is rather late, but I was thinking about contextualism.  It seems like contextualism would be a bad route in deciding upon rules for what is and is not beautiful.  Stecker lists 4 different parts of the contextualist paradigm that basically establish that beauty is determined after long consideration of the work's history and purpose.  To me, it would seem that contextualism requires a lot from the observer.  Rather than there being a solid set of rules to define beauty, one must evaluate on a case-by-case basis and learn mroe about the background.  I think true beauty shouldn't require prior knowledge or history, nor does it need to evoke truths...it just simply as to be something admired and preserved for simply its existence.  It's possible that I am misunderstanding contextualism, but it seems like contextualism is the existentialism of art. 

JB's response. Contextualism is a theory about the properties of a work of art and therefore about the work itself. It is only secondarily about beauty. Its beauty relevance comes from the beauty of an art work depending on what properties it has. A work that seems utterly banal may have meanings that make it far more creative than it seems if you don't get those meanings -- because you don't know the context. Warhol's Brillo Box can't be seen to be an insightful and creative departure from the norm if one doesn't know what the norm is and isn't ready for something new because of not having been sufficiently exposed to traditional art and other aspects of culture.

The idea that beauty should be easily appreciated by everyone has a nice egalitarian ring to it, but it applies only to easy beauties. The aesthetic enterprise is endlessly exploratory, always pushing out the boundaries, looking for aesthetic challenges, things that one's grandfather couldn't have appreciated. Of course one can define beauty in terms of what everyone likes (see my response to #17) but a philosophical theory must cover other forms of beauty that become likeable after the easy forms are mastered. These more advanced or acquired tastes are the result of expanding our sensibilities along natural lines. The easy forms of beauty aren't left behind because it is precisely from the stability they provide that people can venture further. Keep in mind that any beautiful property is a form of excellence.

31. From Alex Culver, 10/20/08, re. Stecker's minimal conception of aesthetic experience. Stecker's minimal conception of the aesthetic experience may leave out certain aesthetic experiences, I think it is a good starting point.  When I think of the minimal conception of the aesthetic experience, it reminds me of hedonistic utilitarianism (what brings the most pleasure is best).  Therefore, I could see that aesthetic experience must qualify as something that gives you pleasure, whether it be audio-visual or on a more intrinsic level.  I think that the two-level philosophy is one way of taking the minimal theory and expanding it.  I imagine that if you started with the minimal theory you could create any number of complexities until you included all aesthetic experiences and excluded everything else.

JB's response. Be sure you explain in your essay what the minimal theory of aesthetic experience does and doesn't include, how it relates to the super-minimal theory (which doesn't require a value component) and how it differs from Levinson's. To be sure, being minimal it includes a lot, more than Levinson's and much more than Bell's or Ingarden's. Levinson's came first and Stecker arrived at his by not requiring two levels. I am skeptical of the minimal theory because I think that aesthetic properties are a lot more pervasive than Stecker does. Second order sensory properties (vividness of color or tone, for instance) are not merely sensory but involve the imagination and cross-categorial resemblances.  

32. From Matt Bakalar, 10/20/08, re. the ontology of aesthetic properties. When discussing the ontology of aesthetic properties, Stecker describes response-dependent realism. In this theory, an aesthetic property is the steady disposition of an object to produce a particular response in an observer. I am having trouble understanding what makes an aesthetic property more "real" than an aesthetic property as described by rational subjectivism. In both cases, certain features of the object  cause a particular response in an observer. Response-dependent realism seems very different from intrinsic property realism, like that expressed by Plato, where a property is the instantiation of a form. I can see how intrinsic property realism posits something real in the object, but I am having trouble with response-dependent realism. Could you please clarify the ontological status of aesthetic properties under response-dependent realism?

JB's response. The key difference is that rational subjectivism only tries to settle on an orderly and constructive set of values for oneself, without any belief in their being best for others or "right." Response-dependent realism takes that extra step and posits a universal value (for humans). The idea is that optimal conditions will produce convergence of the maximal discriminators as to what is beautiful (or has any other aesthetic property). Check the other items in this category (see the table of contents) for further explanations.

33. From Nadia Saleem, 10/21/08, re. drug-induced aesthetic experiences. It seems like there is a great deal of importance placed on the aesthetic reactions of viewers, especially in nature. If these perceptions are so important I wonder what effect different drugs have on the viewer and on the value of the aesthetic object. Greater amounts of estrogen released during adolescence has been thought to cause plumper lips in females. The fact that this has been found to be more aesthetically pleasing is probably due to an evolutionary effect. Does the fact that evolutionary processes are manipulating our reactions lessen the importance of our reactions? Similarly there have been some studies which have found that men with symmetrical and average faces produce better smelling sweat. And that women are more sensitive to this when they are at peak fertility. If these are in fact connected, it could mean that a lot of the emotional reactions we experience are in response to things we're not even consciously aware of. Take it one step farther and consider the effects of depression or bipolar disorders on the audience. It might certainly increase the emotional response. Similarly a hallucinogen would alter perceptions and a mood enhancing drug might make the grand canyon that much more inspiring. There are many ways that people's perceptions of aesthetic beauty can be altered. Is this kind of manipulation allowable in appreciating aesthetic beauty? Is the purposeful use of a drug (such as a hallucinogen) the same as a naturally occuring unintentional use of a drug?

JB's response. On the criterion I proposed it's only cognition-enhancing states that count in maximizing the validity of aesthetic experience. This goes along with the idea of maximal discrimination being the a basic measure of one's aesthetic subtlety. Then there's the factor of convergence of the best discriminators. The best situations for accurate aesthetic discernment will be those that promote more rather than fewer discriminations. Finally there is the factor of non-defective pleasure (error-free, un-pathological, etc.). If all these conditions are met I don't think it matters what biochemical factors are in play. See my response to #16 on the connection of aesthetic sensibility and evolution. As to drugs, I think they are relevant only if they enhance our powers of discrimination and produce non-defective pleasure. Generally they don't. They make us think patterns are better than they are (psychodelic art is a fine proof of that) -- or occasionally worse than they are, even terrifying. They also produce images and patterns that don't exist in the objects or scenes we are viewing. That would count against those states being accurate perceptions. What about the hallucinatory patterns themselves? They might be beautiful but if the drug lowers our powers of aesthetic discrimination we won't be able to tell! What drugs can do is to make one feel that one is having stupendous aesthetic experiences. Again, the psychodelic literature is full of such testimonials. And conceivably those subjective experiences might lead one toward greater aesthetic sensitivity in one's sober states. In that way they could have long-term instrumental aesthetic value.

34. From Nadia Saleem, 10/21/08, re. artistic use of psychological ploys. Is it allowable to make use of psychological manipulations in order to evoke stronger reactions in art? It seems like some sorts of psychological manipulation are used all the time such as in the scary background music found in scary movies.

JB's response. All art, like all rhetoric and human discourse in general depends on psychological reactions. Manipulation occurs only when an artist uses cheap tricks to cover up emptiness or silliness or some other deficiency. Background music needs to be appropriate to the action and the theme of the movie, so that wouldn't count as manipulation unless it's too stereotypical or not well coordinated with the rest of the work. Background music is often a signal, just as lots of other things are in the action or scenery. It could manipulate the audience if it misled the audience. It could annoy if it were overdone, as if the movie-maker thought the audience was dimwitted. The artistic requirement is to use such triggers artfully, appropriately for the intended audience, so as to alert them, produce the appropriate intensity of response for the film -- that is, an intensity justified by the action. This shouldn't be considered manipulation.

35. From Michael Rudman, 10/21/08, re. an alien's aesthetic response to environmental beauty. One idea that comes to mind about knowledge and environmental aesthetics (topic 2): Suppose, putting all knowledge about the world aside, an extra-terrestrial landed on Earth for the first time to spend one year exploring the nature of the planet, I wonder how it would react to the natural environments on this planet. I think overall it would have the ultimate aesthetic experience, seeing leaves on trees bloom, change color, and die, view animals in natural habitats going through different phases of the year, seeing what the sun and moon appear to be like from the surface of the planet, including sunrises, sunsets, and lunar phases. The alien would be in total awe of the inner workings of the planet and would desire nothing more than to know how they worked, like an impressive magical illusion. However, learning the secret to a magical illusion could lead to one of two reactions. Either you become underwhelmed when you see the secret, and lose interest. Or you learn the secret and still take pleasure from seeing the performance of the trick. However, illusions are fabrications of humans and of the human mind. The secrets behind our natural environments go beyond our control. They will continue to go on long after humanity is done with this planet. Like with everything small, however, even with the big things such as the planet, life does not last forever. It starts and ends. There is birth and death with the planet, just like with everything on the planet. It is a cycle of life that exists all over the entire universe. Having that sort of understanding of existence, can only be seen as beautiful.

JB's response. Sure, a newcomer's experience of earthly might be especially intense -- it would probably depend on what experience she had had before. It might also be rather aversive if the alien was used to more beautiful scenery and fauna. But novelty can also be misleading, so we shouldn't assume the alien's experience is truer to the actual beauty than ours. That would depend on who best satisfied the criteria I proposed. And the alien would have to stick around long enough to learn about environmental order if she was to include that in her experience. No small job, that! As to the beauty of the whole, I still think you are being too credulous -- too pious, so to speak. The cycle of life undoubtedly has some beauty but not necessarily the maximum. And we aren't really in a position to deliver a judgment from knowledge about it.

36. From Erin Whiting, 10/21/08, re. Ingarden and the physical aspects of sculpture. Ingarden claims that in appreciating an object aesthetically one must ignore the reality of the actual object (in the case of the Venus the fact that it is a block of marble)in order to appreciate it on another level. But I would argue that an at least subconscious knowledge of what the object was made of and the work required of the artist would enhance appreciation of it. If we actually saw it as a woman and completely ignored that fact that it was once stone, chipped away meticulously until it resembled the female form, we would have no more aesthetic attachment to it than if it were any other woman.

JB's response. I completely agree with you that the physical properties of the sculpture are essential to a proper appreciation of it. And yes, to imagine all that away would give us a different aesthetic experience -- but here please note that it could (conceivably) be a vibrant aesthetic experience, just not of the statue. In reality we can't easily ignore the marble, the chips and scratches and we make the best of all that by appreciating the look of permanence, of resistance to wear and tear, and of pathos, of beauty surviving the wreak of time. Sculptors select their material with an eye toward the effect that it will have on discriminating and imaginative viewers. Ignoring the physical nature (and look) of the work need not be any part of the constitution of an aesthetic object, even on Ingarden's general approach. He overstates his own case in order to emphasize the non-physical additions and enhancements. That's a shame.

37. From Chris Desimone, 10/21/08, re. Ingarden. What objections are directly offered in Ingarden's essay? I have only read it over once and cannot seem to find the link for it again.

JB's response. The extended discussion is in the reserve reading immediately after the Ingarden excerpt. I can't reproduce it here but you can go to ELMS and bring it up. Lectures 13 and 14 contain echoes of it. See especially Lecture 14, section 3 for arguments against objections to the work of art being physical. A key idea is that the work's having intentional properties does not mean the work of art isn't a concrete physical object.

38. From Morris Panitz, 10/21/08, re. the ideal observer criterion. With respect to optimality and the ideal observer, is knowledge of the specific genre of art necessary to make someone an ideal observer? If so, it seems like a cultural bias could be present if only Europeans could analyze Euorpean art, etc?

JB's response. Needing to be well backgrounded is not unfair to outsiders. It's a requirement of all cognitive activity that knowledge of the context is essential. Don't confuse igrnorance with victimhood or knowledge with bias! Why should we expect to be good judges of an art form if we are inexperienced in regard to it?

39. From Megan Maizel, 10/21/08, re. environmental aesthetics. In environmental aesthetics, some believe that in order to truly appreciate the environment, one must have knowledge of the complexity of its systems and the ways in which they are organized.  Our ancestors had a limited or skewed scientific background, but we still like to think that they had some appreciation for the aesthetic.  For example, one can still admire the Earth aesthetically while thinking that it is flat or holding that it was created by a "designer" of sorts.  Given that our knowledge of the Earth is ever-changing and scientific "facts" are constantly being shattered and changed, how does this order view hold up?  That is to say, if we are required to have a deeper understanding of the underlying processes of our world in order to truly appreciate it aesthetically given that our knowledge of such things is always going to be incomplete or inaccurate, can we ever have a true aesthetic appreciation for our environment in this view?

JB's response. Ignorance of lots of sorts is entirely compatible with aesthetic appreciation of many aspects of nature. But it blocks appreciation of the aspects that require knowledge. Obviously appreciation has to be based on something. Perception of sights and sounds enables us to appreciate them. To appreciate specific sorts of order you have to be acquainted with them and with less well ordered alternatives. That's basic. Our distant ancestors appreciated nature in part because of mistaken ideas, ideas concerning mythical beings, spirits friendly and hostile, etc. They were appreciating a fiction with a natural component -- as we appreciate a historical fiction. They weren't appreciating the actual order of nature. I stressed how vast and diverse "nature" is, how much it covers. Of course we can't appreciate many of its aspects even now, but as our knowledge improves we can be more accurate in our appreciation. And of course we can also specialize in the aspects for which our perception and our knowledge is adequate -- no revision of science is going to undermine that. But our appreciation will never be perfectly and accurately comprehensive. That shouldn't be surprising.

40. From Megan Maizel, 10/21//08, re. the ideal observer criterion.  With regard to the ideal observer referred to in response-dependent realism, I am having trouble seeing how an objective observer can accurately represent any kind of aesthetic experience when such experiences are normally thought of on a personal level.  For instance, when I see a beautiful painting or landscape, it invokes personal feelings which ultimately comprise my aesthetic appreciation.  Such general characteristics as its placement, coloring, etc. are only the base of its aesthetic, and to judge something aesthetically by the aggregate of personal feelings towards those characteristics seems to miss the mark in my opinion.  Is the ideal observer the "average" with regard to the myriad personal feelings invoked by a work of art or nature, and if so, how is this even possible given their multitudes and diversity?

JB's response. First, the commonsense notion of aesthetic experiences as merely personal is erroneous. Our perception, our concept-forming intelligence, our feelings and our behavior are shared by all normally competent humans. Of course there are personal idiosyncrasies, but these are mere wrinkles on the great uniformity that binds us into a single species. Training is needed to sharpen and sensitize our powers of discrimination. The ideal observer is someone who scores high in that respect. No actual person is perfectly ideal, but the concept is clear and panels of highly qualified observers in a given art form approach the ideal. There is plenty of evidence of this in culture.

Of course if a work is to be aesthetically rewarding for you, you must also find it personally inspiring, enjoyable, uplifting, whatever. And you can do this without coming close to the ideal. The ideal observer theory doesn't deny that for a moment. Also, the works getting high marks from the experts are also enjoyable, uplifting, satisfying to them on a personal basis. But their pleasure is better, more reliably keyed to the real values of the work than is the pleasure of a poorly backgrounded or casual observer, to say nothing of someone from a different culture. See also my response to other items in this file, esp. ##17 and 19.

41. From John Carter, 8/27/08, re. seeing things in photographs. Stecker, in his discussion of whether "seeing-in" occurs in photographs, says the following: "[I]f seeing-in occurs at all in photographs, we can see-in them kinds of things not represented, just as we can with paintings.  In that case, there will certainly be no such things to be seen."  I'm not entirely sure what he means by this.  You make the comment at this point in the lecture notes that it is "[e]asy to see non-existent things in photos (or in other pictures)".  I had the thought, when Stecker began the discussion of photography, of those visual anomolies that can crop up in photographs (such as the things people interpret as seeing ghosts), and your comment seems to match with that thought.  However, in reading Stecker, I find myself thrown by his wording of "things not represented".  Am I misinterpretting (a) in thinking that if it can be seen in the photograph, it is being represented, and thus (b) in reading Stecker's comment as referring to things that are not appearing in the photograph?  Something isn't meshing here.

JB's response. Here are my first thoughts, which I'll review and maybe update in class Thursday. If we take seeing-in as a certain kind of visual experience then surely whenever we have that experience viewing a picture we are seeing whatever we think we see in the picture -- correctness or truth to the picture is irrelevant. And that goes for photos too. But of course if our seeing-in isn't true to the photo then in a clear sense no such thing is there to be seen in it. The confusion is brought on by "seeing in" having several meanings. In one sense it refers to seeing the subject in the picture -- and if the subject doesn't include what we see-in it then in that subject-seeing sense we only think we see it there. But in the purely experiential sense we do, since we have a seeing-in experience. This difference will be stressed Thursday when I show examples where a picture promotes the seeing-in of things that definitely aren't part of its subject. I personally think this anomalous seeing-in is of great importance in the pictorial art.

42. From Tracy Yau, 10/30/08, re. photographs. As mentioned in today's lecture (10/30) we talked about Walton's theory that photographs are different from pictures because photographs are of things that actually existed and allow us to see the object indirectly and shows us what is there. However, pictures are done by an artist's judgement. I was thinking that pictures and photographs cannot be so clearly defined in this way because photographs are often times framed in certain ways to display the artist's desired intention. If this were the case, it would be hard for us to say that the photograph actually displayed accurate information. The photographer could manipulate the image very simply by taking the picture at a certain angle or zooming into an object or even changing the lighting of the object to make us see something else or believe something else. All these things can be done without photoshopping or editing any part of the image. The photographer displays some sort of judgement when he is taking his photograph just as an artist would display judgement on how s/he would paint the subject. Wouldn't that be why photography falls under the category of art as well? I think I am thinking along the same lines as Lopez.

JB's response. Unquestionably photographers exercise artistic imagination and judgment in selecting the views, the exposure, the lens, sometimes the lighting and sometimes arranging the scene, and in managing the developing and printing. So there's plenty of artistic creativity. No one can look at good art-grade photos without recognizing this. Walton doesn't say or imply otherwise. His point is an ontological one, having nothing to do with the art/nonart question. In viewing a picture we literally see the scene that was photographed, whereas in even the most accurate painting, say a portrait or a landscape, we see only a picture of the subject, not the subject itself -- even if the painting shows the same subject with exactly the properties the photograph shows it to have. Put photograph and painting side by side. The photo enables you to actually see the subject, the painting doesn't, even if it reliably informs you about the subject. The photo is "transparent," the painting is not. It's all a matter of causality. The causal line from subject to the viewer doesn't go through the mentality and handcraft of the artist (absent Photoshop or other fabrication). What is shown is what was there, not what the artist imagined being there or what the artist's hand managed to get down on paper. That's the argument.

43. From Caitlin Dietsche, 11/5/08, re. artistic intentions. Last week, we talked about an artist's intentions and who is the best judge of the artist's true intentions. I was wondering how this argument applies to pieces of art, perhaps of an everyday object like a chair, in which the artist is solely presenting a subject for consideration, with no intentions in mind. Might some artists chose to present their art without intentions?

JB's response. I need more information about the case. Are you talking about found art, as when Duchamp presented a snow shovel or bottle rack as an artwork? He certainly had artistic intentions of a sort, though of a decidedly unconventional sort. Or do you mean a chairmaker presenting her works? Furniture makers certainly have intentions too, only they aren't usually regarded (by furniture makers or the artworld) as artistic ones since the furniture is not thought of as art. Of course there are exceptions, what we could call designer furniture. Here the designer has artistic intentions, even if the "art" in question is generally regarded as a minor or less exalted one than what is usually called fine art. Or finally, do you mean cases where an artist presents something privately for you to look at, not presenting it as art but just as something that is aesthetically interesting? That wouldn't be a case of art unless the artist does something more to make it so. Of course the artist might hope to get an idea from it for a work of art, but that by itself wouldn't affect the status of the object as art/nonart.

It occurs to me that you might have meant that the artist hides her intentions. Did you?

44. From John Carter, 11/6/ 08, re. expressiveness. In his discussion of expressiveness, Stecker has a lot to say about expressiveness in instrumental music, some to say about expressiveness in lyric poetry, but he doesn't really say much about their intersection, lyric music.  What would be the best way to find the expressiveness in music with lyrics?  To adopt the lyric poetry approach, while keeping in mind the emotional contribution of the music?  Or would it have its own distinct way of finding the expressiveness?

JB's response. In many cases the expressiveness of the music harmonizes with that of the words. But not always. Some critics complained that in Gluck's opera Orpheus and Euridice Orpheus' lament is set to music that just sounds tender, not grief-stricken. And regardless of the merits of this particular case, such a discrepancy can happen. The expressiveness of the two can come apart. Sometimes the music may give a new spin to the words, or the words to the music. An ironic touch in the words may affect how you hear the music -- you may hear it as insincere. Interesting and sometimes really subtle possibilities here!

45. From Caitlin Dietsche, 11/25/08, re. Susanne Langer's theory of art. In class you mentioned Suzanne Langer's theories on feeling and form. Her ideas, if I understand them correctly, seem almost biological-- in that they support a biological predisposition to feel aesthetic pleasure from certain things as opposed to others. Could you elaborate on her ideas and explain (if any) the tie between aesthetic experience and the possibility that aesthetic appreciation (or at least feeling and form) is predisposed genetically across the human species?

JB's response. Langer's key idea is that a work of art is a symbol of human feeling, expressing what the artist knows about feeling. The "feeling" in question covers both bodily feelings and introspectible mental states of dynamism (e.g., eagerness, reluctance, hesitation, confidence). It doesn't include the propositional content of an emotion. Feeling states are experienced by all humans, and presumably from the same neurological sources in the body, including of course the brain. The patterns of swelling and contraction of feeling are symbolized by patterns in art works. The environmental triggers of feeling vary somewhat from society to society but there are huge commonalities. Accordingly art forms have the potential to be universally appreciated, even if people unfamiliar with the culture may have to overcome prejudices to seek out the pattern that carries their symbolic significance. Grasping that significance generates cognitive satisfaction on an intuitive plane rather than on a conceptual one. We feel our feeling for life present in the music or drama or dance or poem in a fairly specific form.

There's a connection between what Serra was saying about expansion of perception being the goal of art and Langer's idea of recognizing feeling in art. Serra won't call this a perception of beauty, but the harmony Langer talks about seems to me to be a form of beauty in the all-encompassing sense that I favor.

Langer's idea of patterns of organic feeling does suggest that cognitive science might shed light on both the symbolized and the artistic symbol. Just what the result is remains to be seen. Perhaps when one listens to music the same brain centers will light up as when one undergoes a given feeling.

46. From Brian Morris, 11/30/08, re. essentialism vs. non-essentialism. The Essentialist view is that there is a unique value that makes something art. Where the non-essentialist believes that there are multiple values, which may not be unique to art, that make something a work of art. But can there be more of a middle ground; say instead of there being just one unique value of art that there is a unique combination of values that indicate that something is a work of art? And unlike that non-essentialist, there may be features that are unique to art, or perhaps just more relevant to art?

JB's response. I have suggested that perhaps the distinction is somewhat muddled. No one precisely defines what a unique value comes to, as opposed to multiple values. So there are questions about exactly what the competing positions say. But aside from that, I think there are possibilities of finding some middle ground that is more plausible than either a simple-minded singular value or a messy set of multiple values. However, any true picture of art is going to have to acknowledge a number of differences: serious art vs. entertainment art, major arts vs. minor arts, etc. The whole picture is bound to turn out to be fairly complicated. We'll talk more about this in class.

47. From John Carter, 12/1/08, re. the concept of art. There has been a slight theme running through the past few chapters, little comments here and there by Stecker, Serra's vehement insistance that architecture isn't art, and so on, and I noticed it in one of your distinctions in your answer to Brian Morris' question: "serious art vs. entertainment art".  My question, in response to this particular thread of thought: when is something that looks like/sounds like/otherwise appears to be art, not?  What makes classical music art but popular music not?  What makes the "art house" movies art (beyond having "art" in their description) but the Hollywood blockbusters not?  Related to this particular line of inquiry, it seems like if it's "popular", it's probably not art -- "art" (especially in terms of movies and music) is that which is expected to be limited in its audience.  More to say, in this sense there seems to be an association between "art" and "indie": when something that was "indie" becomes "mainstream", it loses its "artistic creditability".  So, what's the point where a piece of music, for example, stops being "art"?

JB's response. There's no precise point, rather a very fuzzy border. But the paradigms of serious art and the paradigms of "mere entertainment" are clear enough. "Legally Blonde" is entertainment. "Synecdoche, New York" is art (even if not the greatest). Works frequently fall in the border area. I think Stecker's criteria give the right general direction -- I'm referring to the ones on p. 231: originality, aesthetic excellence, and relationship to past works of art (unquestioned cases, that is). The degree to which Gaut's criteria apply is likewise a pretty good indicator (see Lecture 9, Section 4). There are obviously degrees to which a work deserves to be regarded as serious art, art in Serra's sense. So there are preeminent cases, normal cases, borderline cases of a considerable variety of types, and near misses, as well as things that clearly fall into other categories. I'll talk more about this in the lecture today.

48. From Corey Checketts, 12/2/08, re. animal art. The nature of my question for this discussion is addressed towards the idea of animals presenting works of art. Stecker mentions briefly of one artist that uses a goldfish to paint, where the goldfish is dipped in paint and allowed to flop about over a canvas. Similarly, the Gorilla Koko, with a brush placed in hand, can create simple brush strokes on paper which can be presented as art. But are these the relevant intentions of the artist? Are Koko's creations of simple color strokes on paper to be understood in terms of art? Several theorists claim that an art-work has a historical context, artist's intentions and is to be presented in this relevant context with the intentions of being appreciated. Does a work of art have to be created by humans? Or are we to attribute Koko's painting to his trainers who present them as such? If an art-work must be put forward by a human, then what is the reason why Koko's painting is not to be considered art when similar works (Jackson Pollack's splatter works for example) are considered works of art by some. For me I feel as if we should include Koko's paintings under the realm of artistic expression and therefore should be considered as a work of art. But what are the artist's intentions here? Is our appreciation of Koko's work diminished because he is unable to comprehend the magnitude of his work, because he is incapable of careful planning, of the same sort of intentions that can be evaluated with human artists? I say to an extent that our appreciation of Koko's work is diminished because of the lack of background information, the lack of a commentary by the author, and the simplistic nature of the compositions. Yet, one cannot easily say this diminishes our appreciation. We could revel in the fact that these works are created by a non-human, so we cannot assume diminished appreciation just because Koko is an animal; the situation is far more complex.

JB's response. Animal art (all we know) is like children's art only somewhat more limited. As you say, we can stand amazed at the fact that they do as much as they do, but this is a response based on prior ignorance and perhaps prejudice. We should not be surprised at the present stage of animal science. We should say, of course animals share quite a bit with us, and this is one thing they share, up to a point. But the artistic quality is drastically limited, as much accidental as intentional, derivative from their interaction with humans but lacking in many art-making factors. They are not, and they don't have much chance of becoming, members of the human art world in anything like full standing.

49. From John Kim, 12/2/08, re. expressiveness. Stecker, in Chapter 9, briefly denounces the the evocation view of expressiveness in music.  The view claims that "the expressiveness of a work or passage consists in its tendency to evoke emotion in its audience". Stecker's first objection to this view is clear, in that, reactions are highly variable among persons.  However, Stecker's second objection is far more ambiguous.  He claims "even if they were widely shared (in reference to the reactions), the evocation of emotion in an audience is more a reaction to expressiveness than the expressiveness itself.  Hence, it is the wrong item to use to analyize expressiveness."  No more is said in regards to this second objection, and I have been able to envision different interpretations of this objection.  For example, Stecker may be aiming to say that the "the evocation of emotion is a reaction to [the subject's conveyance of] expressiveness than the [state of] expressiveness itself."  Another interpretation may yield that "the evocation of emotion is a reaction to [the state of} expressiveness than [the intrinsic properties of] expressiveness itself."  Could you help clear this up?

JB's response. What Stecker means is that the work's being expressive, say, of sadness may cause the listener to empathize, thus to feel sad. But that causal power (the power to evoke) is not the expressiveness itself, just a response to it. The same is true of a person's expression of sadness. That may evoke sadness in another, but it isn't defined as the power to do so. It would be an expression of sadness even if it didn't evoke sadness in others. That is, if the sad person were surrounded by an unempathic society.

49. From Brian Morris, 12/2/08, re. the concept of art. Following the pluralist view, how would one go about defining art? At what point does one just say that everything has some artistic value? Does the "amount" of artistic value have to surpass something in order for an object to be classified as art? For example it seems that some believe architecture, has some artistic value but is not an artwork. So at what point would a work of architecture have enough artistic value to be a work of art? Or is that even possible?. It seems that there are many blurred lines. The Statue of Liberty seems to be a good example of a work of art with several architectural features or even architecture that is art. But if we consider the Statue of Liberty as an artwork, what separates it from say the Capitol building or a chapel?

JB's response. The Statue of Liberty is an interesting case of a work of sculpture that has architectural features. I call it a sculpture because its interior is decidedly secondary, a matter of practical convenience (for maintenance) and an observation post for tourists. These features are not architecturally significant, so they don't make it architecture. The Capitol on the other hand was designed and built to house the legislature. The fact that it has various features that are in a manner sculptural (e.g., capitals of columns and pilasters) doesn't make it sculpture. Even if had lots of sculptural friezes on the pediments it would be architecture. Some European buildings are richly furnished with external sculptures but are still architecture, not sculptue.

Quality is an important factor in determining whether a given work is art, as Stecker says. I don't think thee's any paradox or impropriety in this. Most basically, an aspiration to create something of quality is what counts, but achieved quality must be the norm. A judgment of quality is especially important in cases where the object is a kind that normally is not counted as art. Exceptional quality can lift it out of its normal category into that of art (the museum quality effect).

50. From Morris Panitz, 12/3/08, re. art and architecture. When we watched the videos in class a week ago about architecture, I was puzzled about one comment in one of the Charlie Rose videos (I forgot who was being interviewed). The interviewee stated that architecture was not art because art should not be practical. Buildings, he argued, are practical and used primarily for purposes other than art. To me, this seems like an odd critique, particularly because it seems to leave the consequence that an architect's drawings or models would be art, but the building itself is not. Could you explain this contention.

JB's response. I don't think Richard Serra thinks the drawings are real art either (or the models, for that matter) because they are dedicated to designing the building, not set forth as self-sufficient artistic works. Serra is focussed (or obsessed) with a more concentrated artistic project in which practicality has no part and no influence. Artists are often like this, so absorbed in their kind of art that they reject anything they consider less pure. I didn't present this video to endorse Serra's view but as an indication of divergent types of art and different levels, so to speak. Sculpture is freer than architecture, and that freedom is both a challenge and a potentiality that architecture can't command -- but of course it has its own distinctive challenges and potentialities. Whether Serra's work meets the sculptural challenge as well as Gehry meets the challenge and fulfills the potentiality proper to museum architecture, is an interesting question.

51. From Corey Checketts, 12/21/08, re. the concept of art. After reading Discussion file #50. From Morris Panitz, 12/3/08, re. art and architecture. He raised an interesting question which I had been thinking of as well regarding that video: the dismissal of something because it is a utilitarian object. What about objects like t-shirts, of shopping bags for that matter. Andy Warhol placed his Campbell's soup can on the surfaces of shopping bags. Sure originally some used these bags for their practical purposes, but some of the bags remain and are displayed as intended-works-of-art. Going back to the idea of the t-shirt. Several t-shirts have famous works of art depicted on the front, others are fresh innovative designs, which possess undeniable aesthetic properties, yet the t-shirt, like the costumed dress you showed in class were not designed as a work of art directly, but also they were designed with a practical utilitarian motive. A shirt is for wearing, but can it also be appreciated as a work of art? What separates and distinguishes say a shirt with the Mona Lisa on it from a replica Mono a Lisa with a mustache drawn on its surface? I suppose this is just a further case of the problem of a definition of art, a difficulty with trying to reveal its essence. For me though, I see these cases, the shirt, the shopping bag, as reasons to support a definition that includes a historical context, an institution, an audience, and intentions, but not necessarily the existence of distinguishing properties.

JB's response. Well, there are important differences of properties between the Warhol original and the T shirt, and these are not trivial. A museum of commercial spin offs would be a real let down from the MOMA! An artist might make T shirts as part of an art project (I know of one who did) but this works only as a part, not as the whole or even the major part, and the whole is hardly a paradigm art work. The T shirt sports a reproduction, for one thing. It's mass produced, for another. And it's worn, and so forth. Sure, it is a collectible (what isn't?) but that's just a symptom of the anal retentive culture we live in. I agree absolutely that intentions are critical, but here the intentions fall way below those of serious art. Duchamp's take-off of the Mona Lisa was a break-away venture in the high artworld, but far from a paradigm of high art. Serra's view is too narrow, but it makes an important point that we shouldn't forget. There is a gradation within the world of high art, to say nothing of all the "after-market" stuff.

Incidentally the costumes of museum quality are miles removed from mass-production goods. They come from highly creative designers for a cost-unlimited clientele bent on having one-of-a-kind originals. Display in haute couture is comparable to the display value of serious art, even if the art is ultimately decorative since it lacks the content to which serious art aspires and in the best cases contains. Nothing made to be worn could have comparable content, not even the robes of state of British monarchs.

52. From Thomas Wilson, 12/4/08, re. efficiency, suitability, etc. as aesthetic qualities. During Tuesday's lecture we discussed how the effectiveness and efficiency with which a practical/useful item accomplishes its intended task is an aesthetic quality, and that such things could be considered art. Does this mean that anything that exhibits a high level of usefulness/efficiency could be considered art? For example, in my studies I have encountered some complex mathematical equations that made 'perfect sense' and solved intricate problems in such a perfect way that I couldn't help but view the equation as a 'beautiful' thing. Could something so practical and logical be considered artistic in this way?

JB's response. I have to agree with Stecker that many things have outstanding aesthetic qualities that are not works of art, even artifacts that embody aesthetic intentions along with their more basic non-aesthetic ones. So your generalization needs to be narrowed down. One additional requirement would be that the function requirements be really difficult to meet if the overall aesthetic merit rests heavily on efficiency. Then the efficiency has a chance of being remarkable enough to be beautiful. A truly ingenious solution to an engineering problem might be beautifully ingenious. Your reference to intricate problems fits this requirement. But there are others too. A mechanism might have nothing to recommend it aesthetically except its ingenuity. That wouldn't be enough merit to count as art even in the wide sense of the term. My idea was rather that a high degree of efficacy or suitability would be a contributor to the art-status of a work that had a fair amount going for it in other ways. This is especially applicable to architecture. Perhaps the amazing marine chronometers made by John Harrison in the early 18th century are a good example, especially the 4th and final one, though the earlier ones are showier. If the Metropolitan Museum of Art could lay its hands on any one of these I bet it would do so, with pride. But really, they have a more appropriate home in the National Maritime Museum in Greenwich (UK). See http://www.portcities.org.uk/london/server/show/conMediaFile.778/John-Harrisons-fourth-marinetimekeeper-H4.html.

53. From Peter Frechette, 12/9/08, re. categories of art. Something which has been occuring to me as we study art is the role of the artist. Specifically, the role of architects, playwrights, and composers. Theirs seems a different art than a painter or sculptor, since their work must either be built or performed to be appreciated. An architect may design a beautiful building, but they cannot build it themselves. Each workman on the project contributes to the realization of the building, but they definitely do not feel that they are making art. Plays must be performed by actors to be viewed as the author intended, but are those actors contributing to the art of the play itself, or are they working on an art of their own, much like a musician? The question is stickiest with architecture, since acting and performing music are mostly considered at least artistic, if not a formal art. However, while the master craftsman of a building may be artistic, it cannot be said that each workman is attempting to create art, when all they are doing is laying the foundation, or framing the walls. Architecture seems too reliant on non-art processes for a building to truly be considered art. (Perhaps the model of the building, or the blueprints might get closer.)

JB's response. The artist in the cases you cite is essentially a designer who relies on performers or workers to realize the design. In the case of painting and carved sculpture the artist does it all, or at least may do it all. The history of these most "autographic" arts is full of partial collaboration, however. Connoisseurs try to distinguish Raphael's hand from that of this or that assistant. Similarly with Rubens, who had a large and extremely high caliber studio (Van Dyck was there for a while, for example). So the difference is not necessarily so uniform as one likes to think. But there is a difference where the composer can't possibly do it all, as in symphonic works, or depends heavily on other performers to spread the work around -- likewise in drama, dance, etc. Architecture depends on a whole range of craftspersons of varying skill and, to a lesser degree, aesthetic sensitivity. Concientious architects do a lot of supervision too -- Frank Lloyd Wright is a good example. This is not essentially different from a sculptor attending carefully to the casting of works or visual artists working in close collaboration with highly skilled printers. In short we have a whole world of relationships that go to make art of different sorts.

I haven't mentioned literary examples, but they exist too: editors, colleagues and intimates who give ideas, suggest changes that are then accepted. And the larger artistic culture shares in the individual's accomplishment, doesn't it? Result: only a quite complicated description of the whole can spell out the manifold types of contribution to creative productions. Nothing wrong with that either. We shouldn't regret that the picture is so complicated.

53. From Caitlin Dietsche, 12/12/08, re. the definition of art. It seems that all our theories about art relevance, experience, value, etc. are dependent on first defining what art is but nobody seems to agree. It does not seem like we can make progress in the field without a universal, accepted definition. What do you think?

JB's response. I don't think we have to have universal agreement about the classification question, what things are art? before we pursue important questions about artistic value, artistic experience, and so forth. If we are sure about what is typical of art we can proceed quite well, even if we won't answer all the questions. One can say what is artistic about traditional art forms even if we may not know what to say about conceptual art or Dada or outsider art, to mention just three examples. I strongly urge people not to be diverted from do-able projects by global reflections about the totality of things. If we clear our minds about the limited projects we have a leg up on the remaining questions. Try it!

54. From Tracy Yau, 12/12/08, re. the nature of art (is art useless?) During lecture we watched the Richard Serra video on art. He mentions that art should be purposely useless. However, if art is purposely useless then I feel that even what we see as traditional art would become non-art. Paintings have the function to capture a landscape or the essence of a person and some are meant for political statements. Does that mean that all of these things are not considered art? Serra's sculptures are meant to convey space and provide area for people to walk in. I think by giving his sculptures meaning he has given his sculptures a function. If I am correct, wouldn't it be fair to say that any object that is made has a function and the use of the adjective "useless" would then become something subjective?

JB's response. One has to interpret Serra's statement, certainly. He's contrasting art, sculpture in particular, with architecture, where the use-factor is massive. Sculpture doesn't have a "practical use" at all in the same league as architecture, which has the purpose of housing things, sheltering people, providing an environment for all sorts of everyday activities. Sculpture doesn't come close to this, not even the big steel spirals that Serra is making. Of course if you push the "use" line far enough art does have uses, as you point out. It serves human functions. But not the everyday practical functions of architecture, technology, industrial craftsmanship, etc. Practical functions, including encoding information about the subjects of paintings, etc., are never the main aim of art. Yielding aesthetic experience, expressing a vision of life, changing human perception (Serra's favorite), etc., are more to the point.

55. From Tracy Yau, 12/12/08, re. architecture as an art. I would also like to add another thought I had during the Serra video. Serra mentions that architects would not say that what they do should be considered an art. However the people he mentions such as Norman Foster is an engineer architect. He is not purely architecture so he designs his buildings from a structural standpoint. Other designers such as Calatrava are also engineers first before they are architects. I feel that Stecker is right when he says that architecture is complicated because there are several dimensions of value and therefore more difficult to give a general rating. There is a mixture of architecture with other functions such as engineering. Would it then be fair to say architecture is not art when the buildings we now see are a collaboration of different aspects? Do you think this is what Stecker means by architecture has "changed" from the past (the past being when architecture was declared one of the 5 arts)?

JB's response. This is a big question. First architecture was placed in a hybrid category in the Diderot's celebrated 18th century Encyclopedia. It was not purely an art of the imagination (fine art), but in part an industrial art (craft, roughly). From ancient times the status of architecture has been elevated by it being very expensive and very important to the state. Architects were managers of lots of workers, disbursers of lots of public moeny, and they were paid accordingly, and allotted a social status in keeping with this. More recently the notion of fine art has been subjected to all sorts of stretching, with avant garde developments of various kinds, as well as opposition to "elitism." The most creative architects are regarded as artists in good standing, regardless of the points Serra makes. Stecker falls into line with this. We could call them the artistic architects, or architectural artists, as opposed to the more workaday architectural practitioners. Still, Serra's challenge is worth exploring, and some of the most artistic of contemporary architects may agree with him about some issues, as he claims Frank Gehry does.

56. From Peter Frechette, 12/15/08, re. expressiveness in music. For expressiveness in music, music is used to elicit some sort of feeling from the audience. This is used to great effect in movies, wherein the music in the background can really affect the audience. Personally, I have had many experiences of feeling sad, scared, heroic, relaxed, etc, based on the music from a movie. Given a sound track to a movie one has never seen, it is easy to identify when there would be suspense, action, romance, tragedy etc. Is this a product of our being accustom to American films and the musical techniques that they employ? Or do we naturally react to music in such a way. I think that some of it must be natural; obviously, high tempo and loud music puts one on edge. Off-key sounds also have a particular effect. But what makes a song heroic, or tragic? Probably, there are elements that we have naturally, and elements that we have learned (by seeing so many movies, and mentally tying the scene to the movie.)

JB's response. Sure, there is a strong natural basis for suspenseful music, triumphant music, and all the rest. This is a matter of cross-categorial resemblances (Davies calls them phenomenal appearances). But that natural basis is supplemented by stereotypes. A particular sort of suspenseful music becomes a standard feature in scenes where suspense is created by the action. That makes the audience response more uniform. I'm sure a careful examination of background music in films would reveal a considerable range of stereotypical and creative expressiveness, with subtleties like ironic contrast and all the rest. Like other elements of movies, the quality of musical expressiveness will vary with the talent and ambition of the movie makers. Some will be art-grade, a lot will be subartistic, so far as the efficacy of the music in relation to the work as a whole goes.

57. From Morris Panitz, 12/15/08, re. the Bible as art. A couple of classes ago, we were discussing whether or not works of philosophy are works of art or produce aesthetic reactions. I was wondering: given the diverse opinions and perspectives on the Bible, how does our discussion on philosophy, literature, and art weigh in on the aesthetic value of the Bible?

JB's response. The Bible is a huge composite text, especially if you include the New Testament. Some parts of it are certainly intended as literary art, e.g., the Song of Songs, the Psalms and perhaps the Book of Job. Many other parts have literary merit and were composed with intentions that figure centrally in literary art, for instance the eloquent expression of important ideas and ideals. The gospel books of the New Testament were chosen from many rivals and perhaps largely on literary principles. So sure, there is much that is artistic about the Bible. But as a whole the Bible was never organized in a literary fashion. Rather religious doctrine compelled the inclusion of a variety of parts that don't fit together in a coherent literary way. It differs from the great religious epics in this respect (the Mahabharata, for example). In a way the Bible is a library of religious texts rather than a single work. And the emphasis falls on edification, on teaching and on proper religious observances rather than artistic exaltation.

58. From Thomas Wilson, 12/17/08, re. aesthetic experience. In Stecker's section on Selfless Absorption, he describes how the respective theories of aesthetic experience of Schopenhauer, Bell, and Goldman each depend on "the separation from the everyday world for absorption in a complex of elements provided by a work of art" (Stecker, 41). From my understanding, Schopenhauer's version is the 'deepest,' as it says that the mind is completely divided from every aspect of human life. However, I am having trouble with some apparent contradictions in Bell's and Goldman's versions. Bell says that we leave all of our worldly emotions and ideas, but he also implies that the viewer must apprehend certain significant-aesthetic-formal features before the transformation of mind can take place. How can we discard all of our knowledge and understanding but still be necessarily attentive to specific, significant forms of the art? Is our discovery of these forms supposed to be accidental? Goldman's version says that we lose all of the practical aspects of ourselves. However, (according to Stecker) we are somehow supposed to maintain enough practical knowledge to allow us to appreciate the historical context and the representative/symbolic properties of the work in question. It seems to me that an understanding of such things would keep us (the viewer) grounded in practicality (that is, I would have trouble being absorbed into (for example) an image of a baseball park if I am constantly thinking about the score of the game and the identities of the players on the field).

JB's response. Bell never goes into detail as to what exactly significant form is. He works mainly by examples and comments about what to avoid. So it can't be said with any precision what goes into experience of it. Clearly knowledge of the form of the object or design is central, so whatever ordinary knowledge of the world is required for that has to be deployed. But the aim is just contemplation, not practical use.That's where the abstraction from ordinary life comes in. In your example if it's the stadium you are contemplating then what's going on in it will be shoved aside, marginalized. So will the weather and anything other than the form of the stadium -- or, and this shows how important it is to specify the object of interest, the more complex form of the stadium filled with spectators and players standing around or running about, etc. Bell never goes into the art of moving objects or processes, but presumably they also have form that is significant or insignificant. Thus an application of his theory to drama, literature or film would have to set forth and assess the formal properties of these temporal artforms. The best way to take Bell's "take nothing from life" mantra is in terms of what is necessary for formal appreciation of whatever subject you are dealing with, realizing that this will vary hugely with the subject. And yes, it will be harder to stick to mere form with some subjects than with others, in general with descriptive and narrative works where we are constantly tempted to empathize or respond intellectually.

59. From Thomas Wilson, 12/17/08, re. aesthetic experience. [not submitted as a discussion question but relevant enough in JB's opinion to require sharing with the class.] In a response to the first exam topic, AESTHETIC EXPERIENCE, it will obviously be necessary to discuss the different conceptions of aesthetic experience mentioned by Stecker and in lectures. However, would it also be prudent to take the response a step further by discussing the different kinds of aesthetic VALUES that can be derived from those conceptions?

JB's response. One way the question of value would be relevant is where aesthetic experience is of little value, as with things too easy to like in an infantile way (cute stuff). Also it seems likely that often aesthetic experience is rather light-weight, compared with other experiences that give more intense pleasure or distress (think of massive physical or emotional pain and the relief therefrom). It is easy to over-value aesthetic experience when one reads the glowing descriptions of it by Bell or Bullough, for example. To a certain extent aesthetic experience in general seems more like icing on the cake than like the nutrition provided (imagine a carrot cake). That's worth reflecting on. As you suggest different theories of aesthetic experience are likely to carry different implications as to the value-potential of the experience.

60. From Thomas Wilson, 12/17/08, re. aesthetic experience. In the transcript of lecture 6, near the end when you talk about your aesthetic interest conception of aesthetic experience, you list the "Dimensions of Aesthetic Experience" which include 'intensity,' 'subtlety,' and 'complexity.' I'm having trouble grasping the nature of these dimensions. Are they sort of like third-level aesthetic descriptors which characterize the aesthetic value of "the instrumental relationship between properties and experiences"(lecture 8). For example, could I say that "the 'complexity' of a scene's potential to produce a variety of enjoyable aesthetic experiences"? I'm having trouble wrapping my head around this.

JB's response. Any experience, aesthetic or moral or intellectual can be more or less intense, more or less complex and more or less subtle. The experience of an aesthetic property can be intense, subtle and complex or weak or simplistic (neither complex nor subtle). Complexity and subtlety will involve nuances of the property itself and so involve the content of the property. Intensity will be a matter of how strong and clear the experience of the property is, however simple or nuanced it may be. Think of the experience of a wine connoisseur who is aware of complex and subtle nuances of flavor and aroma. Or the experience of the vision of life presented by a poet (a la Collingwood) or the experience of a sensitive listener of a musical composition. The various aesthetic properties offer the possibility of experience that is intense, subtle and complex even if most experiencers rarely take in the full measure.

These dimensions of aesthetic experience are recognized by every theory of such experience.

61. From Edison Culver, 12/18/08, re. the concept of art (#54 above). It seemed to me that art could have a function so long as it was also practically impossible to reproduce.  I remember viewing the picture the of sculpted dresser (the 18th c. Newport example) in lecture and its ornate design.  While it has an everyday function, it also seems to stand out in its unique looks.  I would even go so far as to say that art on the wall has a practical everyday use.  I remember going to an art museum in high school and depending on the artist you got a different mood.  I could imagine that art could be used to adjust the mood of those that inhabit a room and be as psychologically useful as background music in movies, etc.  In this way, I am trying to establish some sort of line that differentiates what is considered a "practical" purpose vs. perhaps a perceived purpose.

JB's response. Sure, art works that are made just for viewing or hearing can have, or be given, practical purposes over and above their artistic one. And objects made to satisfy some practical purpose can be embellished artistically. Which purpose is dominant generally determines what category the object is placed in. So furniture however artistically designed and produced is regarded as decorative art, and painting is a contender for fine art even if also used for some practical purpose. There are lots of complications. Painting of stage backgrounds is almost never elevated into fine art, to say nothing of painting on vases. There can be exceptions, but they are placed in a subordinate category of minor (fine) art. The validity of these distinctions, in my view, has to be based on the artistic possibilities of the various types. Artists just about never lavish on stage scenery the intensity of artistic creativity or fineness that they aim for in painting made exclusively for contemplation. Vases don't offer good enough surfaces for ambitious painting. Furniture is likewise so constrained by its practical functions that it doesn't allow an artistically ambitious creator enough scope. All this is perfectly compatible, however, with works of decorative or minor art being artistically better than mediocre works within a major art category. An example of this is the quite fabulous ironwork gates and grilles that are still being produced by the most creative of these designer-fabricators.

62. From Edison Culver, 12/18/08, re. expressiveness of music (# 56 above). Could music, in conjunction with visual elements, say, be given greater meaning by its juxtaposition of elements?  In particular, there are several video directors out there who will use music opposite of the mood portrayed by the video to aesthetically confuse the viewer.  I'm hesitant to give a greater value to this consideration, however, it seems a rather unique way to express a point.  For example, I watched a movie scene where the villain was slaying a family to the tune of "Joy to the World."  Perhaps by using music that contradicts what you see, you end up analyzing the work more and finding more aesthetic properties than if you were to let the video just "wash" over you (like immersion into the environment).

JB's response. The example you give is one of ironic justaposition. Whether it is effective or not will depend heavily on the character of the whole work. The question is always, how much does it contribute to that whole. Does it express a theme that is central to the work, or does it only provide shock value -- one of the most superficial of effects. If it confuses, then probably it's bad, unless the point of the work as a whole is to unsettle the viewer in some constructive, not merely sensationalistic, way. Anytime one encounters something seemingly out of place or excessive, the question arises, what (if anything) is the artistic point of this? But that doesn't insure that the answer is favorable to the work. The idea may be a stupid or lame one. The more excessive, the more urgent is the need of a really good reason. And uniqueness is not by itself a good reason!

63. From Edison Culver, 12/18/08, re. the ideal observer (#38 above). When considering the criteria for the ideal observer, it would seem that art is designed for a very limited audience.  Some philosophers try to find a universal way to explain aesthetics while others develop the idea that only a few people can truly appreciated aesthetic works.  Based on much of the discussion in class, it appears that ideal observers would have to make it their everyday job to understand the backgrounds and analyze art so as to make others understand.  I just want to point out that I don't believe this is far off at all.  It is the reason why everyday citizens aren't dropping their jobs and painting for a living or going around the world to analyze works of art.  It does take a special person to recognize all of the small aesthetic properties that normal observers miss.  So what I wanted to know is whether it is too far reaching an idea to suppose that artists make works of art with the expectation of only certain people to understand?  I could imagine art having the same properties as any other commodity: its value goes up when very few truly understand it. I may just be way off on this one but it would seem that if I were an artist and I made a work that everyone understood it would be quickly viewed and forgotten whereas works of art that are hard to understand remain in the memories of generations of observers trying to "uncover the mystery."

JB's response. Serious (fine) artists try to make art that will fully engage the sensibilities of the best judges. That's one reason for the ideal observer criterion. This doesn't mean that no great work will appeal to practically everyone. Ideally artists would satisfy the best observers and also the public at large. The latter wouldn't so fully (discriminatingly) appreciate the work, but they can be fully convinced of its stratospheric quality. Michelangelo's best works accomplish both goals. The reception of the general public is not a reliable indicator of artistic excellence. That's why no average observer criterion can apply to serious/fine art, or indeed to aesthetic quality in general.

Many writers stress how endlessly rich great art is, how one never seems able to exhaust its qualities, how it keeps on drawing one in, etc. I'm not sure this is quite right, though something like it is. I think one may come to the point of grasping all of a work's properties -- seeing all there is to it -- but just have no way of explaining why these properties work together as well as they do, why the base properties produce the higher level ones, and so forth. But we don't have a deep explanation for all sorts of things in life. The mystery isn't confined to artistic masterpieces. It's found in lesser art works and in countless sorts of non-art.

64. From Brian Morris, 12/18/08, re. aesthetic value. After reading about aesthetic experiences and how other values such as cognitive and moral can contribute to artistic value I have the following questions, Are there certain values that dominate over others? For example does beauty get overshadowed but negative moral value? Or does it all depend of the level of negative value? Suppose that an artwork's beauty is equal to its negative moral value, do we experience any artistic value? Or would we have an aesthetic experience at all? Or is it even possible to think of an aesthetic object as having equally conflicting values?

JB's response. Questions like these are complex. Beauty or value of any sort applies to whole works but also to parts or aspects of works. A work being unbeautiful in one respect doesn't keep it from being beautiful in other respects, and a discerning viewer will make distinctions, praising one aspect and condeming another depending on the merits. How exact a computation of overall beauty (or value) there can be is questionable. So we usually proceed by ranking a work that is, say, aesthetically and morally beautiful above a work that though aesthetically good is morally defective. As you suggest there is room for argument about comparative rankings when both variables differ. When uncertain about either being superior overall to the other we should probably put the two on the same level. After all, it isn't a matter of life or death which comes out ahead. It's much more important to appreciate the merits and limitations of the individual works. And there's no exact bar that a work has to surmount to be artistically valuable. Its values can be artistic even if it doesn't make it into a given rank of art (lowbrow, middlebrow, highbrow). A pop music performance can have notable artistic qualities even if the music (as composed) has little merit.

What is true of art is also true of any aesthetic object (object of aesthetic interest). Things have multiple aspects, some aesthetically good and others otherwise. And don't forget, objects that are aesthetically lackluster or bad are still objects of aesthetic experience -- in that sense "aesthetic objects." Don't be misled by Ingarden's special theory of aesthetic objects. Art objects form a special class of artifacts. Aesthetic objects do not form a special class.

65. From Matthew Bakalar, 12/18/08, re. the ideal observer. Throughout the course, we have referred to the ideal observer, someone who has analyzed numerous works of art and is capable of noticing properties and value of a piece that aren't apparent to the average observer. However, looking at modern art (and the range of "found" art items and variations on "found" art that appear in exhibits today), it seems that the properties that are valued in art have changed. An observer today who viewed a cubist work wouldn't find nearly as much value in it as an observer who viewed the original wave of cubist art (where value is drawn from the incredibly novel way of viewing the world). If this is the case, the ideal observer is not a static figure, but rather depends on the cultural context in which she lives. Doesn't this necessitate taking a contexualist stance on art value?

JB's response. Your idea is interesting, and has some validity so far as the ideal observer not being entirely the same from epoch to epoch. Sure, an ideal observer in the 18th century would lack a lot of knowledge of the course of art history that the ideal observer circa 1950 would have. That knowledge wouldn't matter so far as her response to art work of the time (18th century and preceding several centuries), but it certainly would produce a problem for later art, which we couldn't expect her to understand -- especially the radical new art of the 20th century. But no comparable difficulty exists for later ideal observers, except that an awful lot is asked of them, given the great variation of artistic projects, properties and values. As time goes on the ideal observers have to specialize more and more, to familiarize themselves with their chosen subject -- no one has enough life for it all. As for your supposition about cubism, the later commentators are, I think, better positioned because they aren't so blinded by the novelty of the art. By the same token we are not yet in a good position to judge conceptual art. The test of time, mentioned by Hume, takes time to apply! Cubism is in no danger of being marginalized today. We can see it in proper perspective.

66. From Matthew Bakalar, 12/18/08, re. the concept of art. All human created art is an intentional object, an object created by a person with some motivation. Would the value of an artwork be diminished if it did not have a creator? For instance, could a piece of music created by a computer ever be as beautiful as a piece created by Mozart? What if we were presented with a poem that did not have a creator, but were told that it did. We may attempt to interpret the nonexistent creator's intentions and draw incredible value from our inferences. What consequence should this thought experiment have on the view of art objects as intentional objects?

JB's response. There is no logical impossibility of a computer programed to write English poetry-like texts coming up with something as intelligible and impressive as "Do not go gentle into that good night." Would it be a poem? Would it really have meaning? That depends on the details. Suppose it was programmed to write grammatical (or not too ungrammatical) sentences or partial sentences but was not given any semantic constraints. The vast majority of its output would be complete nonsense, but still on occasion it might produce an intelligible string of words. Within that tiny fraction of its output might even more occasionally be something that looked like a poem. Finally, every once in a blue moon might come "Shall I compare thee to a summer's day..." But in the context of all that nonsense and non-poetic prose why should we take the last to have a meaning at all like Shakespeare's sonnet? Why should we suppose it has any meaning at all? We can imagine it to be poetically meaningful, but that doesn't make it so. Counterfeit money looks just the same but isn't real money, even if fools people. That, at least, is the hard line to take on this topic. We could baptize the computer's output (in this case) a sonnet or whatever, but we'd be giving it the meaning, a very different thing from it having the meaning to begin with. When we give it meaning we treat it the way artists treat found objects, which weren't art until they made them so by baptism (according to some sort of recognizable artist program, not from mere caprice). Result, in my view is that the intentional factor in art is not undercut by the logical possibility you point to. Not everyone agrees with me.

67. From Matthew Bakalar, 12/18/08, re. categories of art. Certain varieties of art are valued more for their aesthetic values than their cognitive values. For instance, while literature draws its worth largely from cognitive value, music, which is capable of only limited expressiveness, seems to be valued for its aesthetic value. Still, it is the aesthetic value of art objects that unifies them. We do not call objects that have significant cognitive value, but no aesthetic value, art (for instance, philosophical works, as discussed in class). Would an object with purely aesthetic value, even exceptional aesthetic value, but no cognitive value, be considered art? If we take a constructivist approach to the question, there can be no such object. The observer of an art object will create cognitive value in any aesthetic object. What are your thoughts on this issue?

JB's response. Several issues here. Stecker thinks that some works of visual art have little or no aesthetic value. Some found objects, and some conceptual art, are examples. True, aesthetic value is as pervasive in art as any value, perhaps moreso than any other sort of value, but still that doesn't make an absolutely necessary condition, in his view. The question of art with no cognitive value isn't much discussed, but I think the issue may be one of how important that value is rather than whether there could be an absolute and utter absence. Probably people simply dismiss the cognitive value of instrumental music or non-figurative painting or pots because they don't think the cognitive value is large enough to be worth mentioning. I think there is always some. At a minimum one gains knowledge of musical possibilities from instrumental music and design possibilities from non-figurative art. The problem clearly needs to be more sharply defined. If you put the problem in terms of how important the knowledge is, then a lot of popular art is disqualified along with a lot of instrumental music and abstract art. I guess your idea re. constructivism is that if anyone whips up an interpretation of a work, she will make something significant of it. Maybe, but then constructivists wouldn't bother with a lot of art in the low-grade, undemanding sense. Constructivism is a lot like found art, as in my previous comment.

68. From Corey Checketts, 12/18/08, re. interpretation. I am going to be commenting on the poem you showed us in class last week consisting in nothing but last names. Does this imply that art is subject most to interpretation? If we can find some sort of cognitive value out of the list of names, piecing together some interpretative meaning, then the work can be said to be poetic, or said to be artistic. In the example given the students found a religious interpretation, and you could say the work has some sort of meaning to it. I feel though like extending this to the realm of art is problematic for a few reasons. It blurs the lines between art and nonart even further. If this list of names is art, then why not a straight line, why not a list of grocery, or some other mundane item? Our common intuition (at least mine) wants there to be some sort of separation between art and nonart works. I think this example creates a problem if accepted as art.

JB's response. What can I say but persevere! No, my position is that Stanley Fish, who pushed the students into finding poetic substance in that list, is just pulling a stunt, like a philosopher who ignores what poetry really is. He might very well do the same with a grocery list. As I said in class, one huge absence is the sonic aspect of poetry. Another is that the interpretation makes a cypher out of the word sequence -- though sidestepping the name that doesn't fit with the decoding. The thing doesn't even have the special kind of kookiness of Dada concrete poetry. In fact it has no relevant context at all, no rationale, no poetic sense. The real problem is that there is no way to make that string of words really poetic. So consign it to the dustbin.

69. From Daniel Eliezar, 12/18/08, re. architecture and art. Here is a thought I had on the topic of architecture: One of the arguments against the idea of architecture being a form of art is that its main function, and intention, is to provide some kind of shelter, working space, etc. Thus, the architect is concerned with its utility as a functioning space, and in doing so he/she can not create art for the sake of art where there are no restrictions on the creative process. But what about art pieces placed in museums? I imagine many of them were created with the intention of being placed in museums. That is, they function in many ways: to be looked at, to please onlookers, to gain revenue for the artist, etc. So is the only "art for art's sake" pieces that are unobserved and outside of museums? In that case, what about an artist who makes a building that can not be entered? It may be an otherwise ordinary building but it serves no function, as the architect intended.

JB's response. See my responses to ##54-55, 61. All classes of artifacts are defined in terms of purposes, art forms included. The essential point concerns practical purposes and the purpose that defines the type of artifact. Art is not defined in terms of practical purposes. Practical purposes are not the central ones in assessing the art as a good example of its kind. Of course "practical" needs to be defined too! Enjoyment of aesthetic properties is not "practical" even though it may have practical benefits. There are also all sorts of practical limits that affect the properties of given works. Portable works have to be lighter than the Egyptian pyramids. Paintings have to be stable enough to keep their colors while the painter is still working. Etc

70. From Daniel Eliezar, 12/18/08, re. aesthetic experience of literature. If we take a piece of literature to be a form of art, how should we consider the aesthetic experiences of reading it? wouldn't each experience be contingent on past experiences? When you read the literature from where you left off from your last sitting, is it a continuation of your last aesthetic experience? In this case, you likely return with a different mood, set of knowledge/experiences, etc. So is the only ideal aesthetic experience, and only way to make a conclusive judgment of the literature's entirety, reading it in one sitting? Or do multiple sittings count as separate experiences? In which case, isn't each sitting contingent on the previous one?

JB's response. Poe argued that continuity of the literary experience was optimal, so short stories were the best literary form. That was self-serving since his favored genre was short stories. Experience comes in phases and also in gap-transcending forms. There's no way to count experiences definitively (how many experiences have I had in the last ten minutes?). So I think we have to be relaxed about aesthetic experience being one or many. Sure, we have momentary experiences in literature or music, and we have resumed aesthetic experience of War and Peace when we pick it up again and get ourselves back in the swing of it. All temporal art depends on memory, but so do paintings and sculptures. One's mood varies in one's experience of all these art forms. One tests the work in part this way. Does it appeal only when I am sad? Then something's wrong with it! Or perhaps with me. The experience that counts most is the cumulative experience of a work, or if you like, the repeatable experiences. All art allows for that (though differently for performing arts when performances are few and far between).

71. From Daniel Eliezar, 12/18/08, re. categories of art. I have trouble understanding the distinction between "entertaining" and "high" arts. This seems to depend on the context and perspective. The tap dancer, for instance, may be presented for entertainment. The intent of dancing may very well be for entertainment. But to other tap dancers, I imagine they would not be entertained in the same respect. They would have the experience and perspective to admire or criticize the performance and the aesthetic material that comes with it. The distinction seems arbitrary.

JB's response. Of course it is essential to be fair to the different arts. Still, some art forms began by being popular entertainment without strong or high artistic aspirations -- and that means they didn't try to be deep or challenging but just enjoyable without the audience having to make any real effort. There is also the factor of the audience not being called upon to stretch its conception of life or its imagination or its perception. Among tap dancers I guess Fred Astaire (who is also an accomplished ballroom dancer) is as good as they get. He's wonderfully suave and agile and graceful, but no one, he included, would call his performances profound. They are meant to be entertaining. So that seems to put a ceiling on the artistic aspirations of tap dancing. Maybe some tap dancer can break through that ceiling by tap dancing more deeply expressive. Fine, let him or her do it and then the new art form will stand nearer serious/high art. Is that so hard to understand and accept?

72. From Peter Frechette, 12/18/08, re. aesthetic experience. There seems to be an element of "knowing what to look for" in AE. This raises the question of natural versus trained Aesthetic Appreciation. How can many people, untrained in the art of appreciating sunset, think that sunsets are beautiful, while the same number of people do not see the beauty in a dog, for example, whereas a small number of trained observes can see the distinctive A. features, and see the dog as beautiful. There seem to be different A. features, those which naturally occur, and those which are trained. Discerning A. features seems integral in making beauty judgments, without A. features, any combination of colors/sounds would be the same as any other. Somehow, particular combinations of A. features are beautiful, sometimes to everyone, and sometimes to just those who can distinguish them. It also seems that A. features can change over time and between cultures. For this reason, a pale, volumptous female has A. features of eroticism in one time and culture, while a suntanned, whip-thin female has erotic A. features in another. We seem to make judgments about A. features as well as AE as a whole. And our judgments on A. features seem to affect our judgments of AE

JB's response. Here's a pretty tangle! But really, it's not so baffling. Of course some aesthetic appeal is so wide-spread and natural that no training is needed. Complementary colors reinforce each other's vivacity and everyone can appreciate that. Patterns that are easy to read are favored over ones that are impossible. And so forth. As we gain experience of life we discern expressive correspondences, our imagination is exercised in a rewarding way, we empathize with the body language of designs, we dance to dancible music, we find subtleties, and so forth and so forth. Smooth unblemshed full skin and well developed curves are sexy, but so are adventurous-looking tanned, lithe bodies. They can provide different but types of pleasures. So with the many, many varieties of aesthetic ensembles. Cultures specialize but open-minded folks can appreciate the aesthetic tastes of others. It's not really all that hard, given the opportunity to learn what it is that the alien culture sees in the forms it relishes. If you make your questions more specific, tied to particular cases, I think the difficulties begin to vanish. But it takes real experience and not just the idea of experience, to do it.

73. From Mike Gabaly, 12/18/08, re. the concept of art. Levinson's main idea regarding his intentional-historical definition states, " something is a work of art because of a relation it bears to earlier works, which are in turn art because of a relation they bear to sill earlier works and so on." Does this mean that Levinson believes for the most part that there are no unique forms of artistic expression? Understandably most everything is a derivative of some other idea or expression, but what would it take for a contemprary artist to truly break free and create "first art"?

JB's response. No, there can be interesting departures from past art without the relation being absent, as Levinson is careful to point out. New advances are often art only because of their rejection of most past values. Yet the sort of regard that counts is one that can be seen as a creative development within the larger stream of art. He doesn't mean all art has to be "derivative," which is a term of belittlement in artspeak. Duchamp is an example of a radically avant garde artist. Yet he retains important links with past art. Without those links Fountain could never be regarded as art.

74. From Peter Frechette, 12/18/08, re. expressiveness. On the question of expression, things get complicated when artists try to express complex emotions. It seems things such as fear, exultation, etc. are easy to express, since they have 'tones', one might say, that correlate to the music or poetry. Fear, for example, is fast, loud, like a heart beating fast. Other emotions are difficult. How does an artist express hopefulness, or hopelessness?  The fact that these emotions can be heard in music, and the fact that they are difficult to produce indicates to me that some sort of intentionalism must be a part of expressiveness. The ideal audience must use the conventions/contexts available to them to decide what the author intended to express. Sometimes this ocurs rapidly, during the hearing-in process. Sometimes, it takes a more carefu examination.

JB's response. Instrumental music doesn't have the capacity to express propositional meanings -- or not very much of it. It can't express specific sorts of hope, such as hope of rescue or hope of business success. But it can express the feeling side of hope, the uplift, the rising spirits, etc. Intention is a hotly contested aspect of this subject -- check the outlines for the relevant lectures. The audience has to use varied cues to decide what the work means, which is normally what the composer intended it to mean, but there are exceptions, real ones and imaginable ones. Yes, of course, often careful examination is needed.

75. From Mike Gabaly, 12/18/08, re. the concept of art (Levinson's). This is a question mainly for clarification, but Levinson responds to his first objection regarding Duchamp's attempt at turning a Woolworth's into a ready made states, "Duchamp failed because he lacked the relevant proprietary right to the building." This is the only justification he gives for this objection by Carney. Can you elaborate on exactly what this means? It seems contradictory to his claim regarding the Rembrandt forger in the next objection.

JB's response. The cases are very different in that Duchamp wanted to change something that is not a work of art into art (of a non-architectural sort!). Levinson says that requires owning the building -- you can't turn my building into your art. Duchamp bought the snow shovel and the urinal and displayed them as art with titles appropriate to them in an art context. He couldn't do that with the building. The forger produces a copy of Rembrandt's painting and tries to palm it off as a Rembrandt. He doesn't represent it as his work. He owns it (in a fashion) and could present it as his art if he liked, but people would laugh at him.

76. From Chris DeSimone, 12/18/08, re. rarity as an aesthetic or artistic value. Can you offer up one reason for justly thinking that the rareness of a piece of art would improve its artistic value. I can't seem to come up with any reasons that don't seem to hold up under scrutiny, or maybe just an example of this being the case may help.

JB's response. No, I don't think rarity makes a work better artistically. But it certainly makes more highly prized among collectors, museum directors, and the viewing public. And why not? To have something rare and valuable is to have access to a valuable thing that others don't have access to.

77. From Keegan McClure, 12/18/08, re. aesthetic value . Stecker claims that by using the minimal view, we can identify two kinds of aesthetic value: intrinsic and instrumental. Since the minimal belief allows us to "remain agnostic" (believe nothing about existence) with respect to aesthetic properties, thus concluding that no special subset of forms, qualities, or meaningful features are required to talk about aesthetic experience, are the "ingredients" that aesthetic experiences have simply their intrinsic or instrumental values? More plainly, is the noticing of forms, qualities, and meaningful features a intrinsic value of an object/expereince, and is enjoying (or noticing) those forms, qualities, and meaningful features an instrumental value of of an object/experience?  If not, can you eloborate at all on what you mean by "ingredients"?

JB's response. If there are no real aesthetic properties, there are subjective impressions of such properties -- as if properties, we could call them. They take the place of true aesthetic properties in our aesthetic experience and in our aesthetic evaluations. There will simply be no rational basis for reaching agreement about the objects of our experience. But otherwise our appreciation is unaffected.

78. From Keegan McClure, 12/18/08, re. aesthetic experience.  Since part of Topic 1 asks specifically about Levinson, I ask for some clarification with respect to the distinction between Levinson and Stecker.  Stecker's minimal view assumes that aesthetic experience requries a special kind of engagement with an object in which the object, some of its properties, or the experience of these is valued for its own sake.  Levinson's view, like Stecker's, includes forms, qualities, and meanings as objects of aesthetic experience, but also calls on the viewer to "attend to the way all such things emerge from a particular set of low level perceptual features".  Stecker seems to object to Levinson's view (via the sunset example) by questioning what the viewer should do when there does not seem to be lower-level perceptual features from which the experience arises. He claims that this objection leaves Levinson's view open to an equivocation or a counterexample, but I do not see exactly why, can you help?

JB's response. Stecker's view is wider than Levinson's (less demanding) because it doesn't require such a structured experience. Levinson's view, he thinks, is defeated by the example of the sunset where there is no need for awareness of higher level properties emerging out of the colors and shapes. That's a counter-example. I think by equivocation Stecker means Levinson might adopt a disjunctive definition, which would go against the spirit of his two-level unified definition.

79. From Keegan McClure, 12/18/08, re. aesthetic experience.  Where you ask about experiences of disfavor, and whether or not they can be aesthetic experiences, are you looking for an answer relating to negative experience that you enjoy for reasons not-intrinsic to them?  The example of attending an exhibit of a bisected cow seems to me a good example of an experince of disfavor.  So, if, upon leaving, you say, "I hated being there, but I'm better off for it. I can deal better with things that disgust me", you would be finding a postive value of the experience (a payoff intrinsic to the experience), thus making the disfavorable experience an aesthetic experiene according to Stecker's minimal view, correct?

JB's response. I personally don't think aesthetic experiences have to be positive, and particularly don't have to be enjoyable or satisfying. They may be unredeemable negative. But sure, often a negative response may be followed by an appreciation that there's a silver lining. I also think some can be entirely neutral about value, but that the experiences we mainly seek are ones we positively value. Stecker favors the traditional view of aesthetic experience being one of positively valuing the object -- or at the very least having some valuational stance toward it (not being purely content-oriented). But note that he says he thinks there is no uniquely correct conception of aesthetic experience. In the end his preferencet comes down to a matter of not wanting to be revisionary when there is no compelling need to be.

80. From Kesun Lee, 12/18/08, re. aesthetic experience. About Stecker's Account of aesthetic experience: As I finished Stecker's book, I can't help but feel as though his account of aesthetic experience is a bit limiting. I feel any experience where the viewer has some kind of reaction, any kind of reaction really, can be counted as an aesthetic experience. By this I mean, when art is physically or mentally viewed by someone, whether it be a painting, a sculpture, a musical piece, a ballet, or a poem, and that viewer has some kind of reaction to this art, an aesthetic experience is had. I find his account of aesthetic experience makes it a little harder to define something as an aesthetic experience. I feel this is inaccurate because not everyone will react to something in a supposed way; however this should not take away from the experience being had. These types of experiences should still qualify for aesthetic ones.

JB's response. No theory can hold that every response is aesthetic. If a very bright light hurts my eyes that's not an aesthetic experience. If a picture reminds me of my home, that needn't be an aesthetic experience. So please tighten up your thinking on that score. Stecker's view is very wide, so it includes almost all candidate experiences. I think it would be better to widen it a bit further. See my response to #79.

Subjective variations in response is an entirely separate question. Sure, experiences don't have to be veridical to be aesthetic ones; they can even be hallucinations. No theory requires otherwise. The question of correctness only arises when there's a claim that the experience represents the thing as it is, or that the thing has a certain aesthetic value.

81. From Mike Gabaly, 12/18/08, re. the concept of art. I would like clarification on the objection to Gaut's cluster conception. His definition states that several different sets of properties can lead to art status but no one specifically can reach said status on its own. The objection claims that because the conditions are "finite and enumerable" they are already equivalent to a definition of art. This is the confusing part. If it is saying that a part of the cluster can be labeled art, why are the one concept definitions (namely Bell's) incorrect? Hopefully you can shed some light on this for me.

JB's response. The cluster conception is put forward as an alternative to a classic definition via necessary and sufficient conditions. But if the set of conditions in the cluster is finite and enumerable, then the conception seems to reduce to a classic definition of the form Being art = Being A or B or C...or N. That disjunctive condition is both necessary and sufficient. Only if the disjunction is open or indefinite can the conception be a true alternative.

82. From Morris Panitz, 12/19/08, re. aesthetic experience. In preparing for the essay on aesthetic experience, I am having trouble differentiating between the content-directed conception and the aesthetic property conception. Could you clarify the difference please.

JB's response. The only difference would be that the content in question needn't all be aesthetic properties. Recall that Stecker expresses some skepticism about the reality of these properties. A content-oriented conception might do so too. See my response to #77 about experience without (real) aesthetic properties.

83. Nathaniel Snyder, 12/19/08, re. artistic value without aesthetic value. I would like to think an office portrait (in this case, of an ex-Governor) might fill the bill. While many famous portraits have reached aesthetic value, I think ones such as this generally only entail artistic value in the rendering of the subjects likeness to canvas. However, as an interesting side note, take a look at the picture from this NY Times cover page, I think it might be to your liking! http://cityroom.blogs.nytimes.com/2008/12/03/create-your-own-governors-portrait/?scp=1&sq=Cover%20Crumb%20Picasso%20Gov&st=cse

JB's response. There's always a vagueness when people speak of aesthetic value. The threshold is never specified. I don't think the threshold is important, only the rank ordering of greater or less aesthetic value. Think of the situation where that's the only portrait we have left. We'd soon find some aesthetic value in it! Aesthetic value comes in many different aspects of things. To say something has some aesthetic value is to say practically nothing. What counts is when a thing has remarkable or impressive aesthetic value. Artistic value is different. I can understand people (especially serious artists) saying a workaday portrait is not art at all because it doesn't have enough artistic value.

The portraits in the Times contest will be interesting to see. I'll have to search for the results in the Times archive. Of the four by Fuchs of Cuomo the one supposed to be like a Picasso is no good at all -- not Picassoid by any measure. The Mondrian-esque one is clever. The other two are not very interesting because Crumb and Warhol make it too easy for the artist.

84. Nathaniel Snyder, 12/19/08, re. Artistic Value of Materials used in art. I want to use as an example, "Portrait of Adele Bloch-Bauer I" by Klimt. This (1st or 2'd most expensive, in terms of sale price, painting ever?) is painted in oil, silver, and gold. I think that much of it's artistic value is achieved through the use of gold and silver, which gives the painting a distinctive quality not observed with simply gold or silver paint. Originally I wanted to use rare gems in exquisite jewelry, but I thought this example would be susceptible to the argument that it constitutes craft, not necessarily art. My question is does this example show that expensive materials contribute to the artistic value, and more specifically, does the use of these materials create their own value or do they simply add to a value that is already there? For example, if the Klimt used only gold or silver paint, rather than the material itself, would all the artistic values still be present, but of lesser magnitude? Then how about substitution of diamond for a lesser stone (of less radiance)?

JB's response. In early cultures costliness of material was highly prized. Nowadays it is not prized at all so far as artistic standing is concerned. Klimt and others who use luxury materials do so for expressive effect and by way of allusion to earlier art, not because they think the quality of the materials in and of itself adds to the artistic value. This is the short account.

But there is more. Especially when art was in the service of religion often nothing less than the finest materials were deemed suitable. Insofar as this suitability was an essential property for artists and patrons, it counted as an artistic value. Luxury arts (called sumptuary arts) were also given top ratings in aristocratic cultures. Artists were paid accordingly, too. Metalworkers were more respected than painters, bronze sculptors more than stone carvers, and so forth. The history of all this is quite complex. Nowadays the artworld looks back somewhat patronizingly on these early cultures as having an impure notion of (serious) art. Such views produce accounts of "our" notion of art as dating only from the 18th century. I find this an oversimplification. Luxury materials were part and parcel of the artistic practice in early times, the most skilful and creative artists used them; they also produced more splendid aesthetic effects than lesser materials and they stood up better over time. The use of lapis lazuli by painters for blue (Vermeer's blue) is a case of this. I think the key criterion for luxury materials being aesthetically relevant is found here, and in the aesthetic appropriateness of the materials to the overall project (as when gold backgrounds in painting or mosaics effectively represented the saints standing in God's "holy fire."). Sometimes luxury material and the workmanship that was required limit the quality of the work overall. I think this is the case for tabletop pictures in marble, agate, etc, that were produced in the15-18th centuries. They aren't all that great as pictorial representations, so they count as decorative, not as fine, art. But they are stunning in their category.

So the subject of expensiveness of materials is by no means a simple one!

85. Nathaniel Snyder, 12/19/08, re. Art and morality. Moral art should promote virtue over vice, but many examples of tragedies, where a single flaw in an otherwise honorable human leads to disastrous downfall, pose a problem to this assertion. Should art reflect that humans are inherently flawed? Should human imperfections doom us? Is it absolutely necessary for moral art to promote virtue over vice, or can't it be enough to simply make us ponder on morals?

JB's response. There's no conflict here. Tragedies teach us moral lessons. They may also teach us tolerance, in recognition of the errors even reasonably well intentioned people can commit, or errors that come from bad social conditioning, etc. To ponder seriously on these circumstances is facilitated by grim stories. The important thing is that the artist or author not espouse immorality.

86. From Nadial Saleem, 12/19/08, re. the concept of art. According to Stecker's Historical Functionalism Theory, artifacts outside the central art form have to meet a higher standard in order to be considered art. How does the property of fitting into established guidelines of art relate to the aesthetic value of a piece? It doesn't even seem to be a property like the properties that we have talked about.

JB's response. It isn't an aesthetic property, to be sure, or not one of the sort commonly counted when assessing aesthetic value of a work. Stecker doesn't think that aesthetic value is all there is to being art. If a work falls within a central art form it participates in a cultural practice. That doesn't make it more valuable overall as a thing of its kind. It may be a lousy work in that art form. But it makes it easier for people to deal with, to understand and appreciate -- and to do that collectively. And there is a value in that, certainly, even an aesthetic value (suitability or intelligibility within a humanly constructive practice). You could think that works within a central art form are given an unfair advantage. But look at it the opposite way. Works outside of any central art form that count as art are ranked above the mediocre works within those conventional art forms.

87. From Nadial Saleem, 12/19/08, re. the concept of art. In determining the difference between works of art and other categories of aesthetic value, the idea keeps coming up of whether or not the objects have utility, a purpose. For this reason buildings are often not viewed to be art. How do you determine what aspects of the building are part of the building and what are decoration? The paintings on the ceiling of the Sistine Chapel presumably would count as works of art. Is this because it's decoration? How do you establish which things are decoration and which things are part of a utilitarian object?

JB's response. Wall or ceiling paintings are judged serious art or decoration by the ambitions and creativity the artist displays. And Michelangelo displays the highest level of artistic motivation (and accomplishment) in his works in the Sistine. On a lower level earlier artists (late 15th c.) do their part too. On the first level Raphael's tapestries were installed when the Vatican pulled out all the stops. So it was a feast of superb art. True, art historians speak of these as decorations. But they do not mean decorative art! In various ways all these art works had purposes connected with the Papacy. But what I've said in response to other discussion items puts this into perspective. See ##54-55, 61, 69.

88. From Nadial Saleem, 12/19/08, re. the concept of art. Where do monuments fit in in terms of architecture and sculpture? Do they count as art? They don't seem to have a purpose in the same way as architecture does... especially not ones like the Eiffel Tower. However, other monuments like the lincoln memorial become more architectural in nature. Even though they still don't serve a very useful purpose such as providing housing or anything.

JB's response. Public monuments are an extremely varied lot. Some are simply sculptures, though commonly with large and elaborate bases. Some are architecture, as in the Lincoln and Jefferson Memorials, which fall into the category of (architectural) shrines. Some come into a category of "structural art," a term that appears in the informative Wikipedia article about the Eiffel Tower. Bridges and other structures would be other examples. Since the majority of these structures are not artistically motivated, Stecker's distinction between an art medium and an art form is relevant -- whether or not one ultimately agrees with his judgments about what is and isn't art in one sense or another. I am impressed by how many different categories of art (in the low or high sense) there are.

The Eiffel Tower has two restaurants, elevators and obsevation platforms, all of which are standard features of buildings. Yet mostly it is, and was meant to be, a huge eye-catcher, in that sense environmental art (urban subcategory). Is it serious art-grade? Lots of contemporary critics didn't think so. See the Wikipedia article. Interestingly, it was planned to make demolition easy, because the plan was to disassemble it after a short while. It was the entrance to an international exposition. But history intervened and it has become an icon of the City and of France. I think it should be regarded mainly as an urban ornament with symbolic overtones and architectural features, hence an eminent work in a fairly minor, hybrid, art category. Doubtless many would disagree.

89. From John Kim, 12/19/08, re. art and architecture. When it is said that architecture which belongs to the art form belongs to the medium, but not vice versa, how can it be that this "necessary but not sufficient condition" is seemingly an exclusive feature to architecture?  It seems that for architecture to have this exclusivity is a sort of implication to the contrary, that architecture cannot have an exclusive feature - almost no other descriptive feature exists exclusive one type of art.

JB's response. This can all be cleared up. The medium is wider than the art form. This is true for all the arts, for painting and sculpture and literature and music, etc, so it's not exclusive to architecture. To belong to the medium is not necessarily to belong to the art form, according to Stecker. I think that there is a segment of architectural practice that is an art form, but many buildings don't qualify as instances since too little artistic motivation goes into them. Stecker prefers to say there is no architectural art form, only individual buildings that are aesthetically good enough to be counted as art. His reasoning does not convince me.

90. From John Kim, 12/19/08, re. art and architecture. The "function of creating an artificial environment connected with a larger environment" - this is the definition of art in architecture according to Stecker.  When objectively judging the artistic value of a work of architecture - does this apply perpetually to parts of the architecture as well, and does this add to the value of the art?  I.e. the structure/support design in relation to the architectural building as a whole, the interior design in relation to the building, designed placement of certain rooms/furnitures in relation to the floorplans/rooms.

JB's response. That's not a definition of art in architecture, it's only a necessary condition in any proper definition. The other aspects you mention (the layout, etc.) are parts of the "environment." There are also decorative features inside as well as outside. All those are relevant, as well as yet others. For the relation of architecture to art, see the previous discussion item.

91. From Megan Meizel, 12/19/08, re. the concept of art. It is mentioned by Stecker that objects which are not considered to be art may possess artistic qualities, such as appliances and furniture.  However, what is to prevent these objects from being works of art?  Is it merely the intentions of the creator (in this case, the intent being perhaps a combination of function and form)? And if there is just one single artistic intention to the making of the object (for example the aesthetic appeal of a particular sofa), could we not justifiably qualify these objects as works of art despite their (presumably) first-order intent of function?

JB's response. It's not just the intentions, though they are important. It's the way the objects are treated, appreciated, evaluated, etc. and that's connected with the properties they have. Object like the ones you mention fall into design rather than centrally into (serious) art because the limited aesthetic properties they have. I've discussed this in earlier discussion items.

92. From Megan Meizel, 12/19/08, re. interpretation. How should we define "convention" in relation to interpreting a work of art? If an artwork's meaning can be inferred from convention and context alone as some claim it can be, what qualifies as the norm?  Should we create a hypothetical audience by which to judge the correct interpretation of an artwork and if so, what are the qualifications of this audience?  Are they specific to the artwork, its creator, or both?

JB's response. Not surprisingly theorists differ about these matters. There is, however, considerable support for the criterion of the well-informed audience's judgment as to the conventions in use. There can also be a criterion using a statistical survey of actual critical practice. Your other questions are covered by the text material -- e.g., qualifications -- the basic idea is that yes, the qualifications do include knowledge of the medium and genre of the artwork and of the artistic purposes current within the artist's circle.

93. From Megan Meizel, 12/19/08, re. interpretation. If a work of art is analyzed and appraised by one who knows nothing about its context or background, is it possible that the inferences made by one examining it under such a different context are necessarily wrong? For example, if I were to derive a meaning from a painting which differs completely from the creator's intentions, am I missing the artistic value of it altogether or perhaps creating my own?  And is to create one's own meaning independent of the creator's intentions to diminish its artistic value as a whole?  Or does this simply reflect the need for an ideal audience in interpreting artwork?

JB's response. If one is after the artistic meaning of the work, then yes, you are missing it if your interpretation is far from that of the ideal audience's -- which is almost without exception also the meaning of the artist or very close to it. If you create your own meaning you are playing a game with the work -- nothing intrinsically wrong with that -- or you are expropriating the work as one might appropriate a stone or piece of driftwood and making a "found object" artwork of your own. That's OK too, in a way. Why not? But neither of these enterprises is engaging with the meaning or value of the artist's work. The popular rhetoric of each person creating the meaning of art works is completely out of sync with the serious art world. No one would seriously devote her life to art and not care whether her artistic point is successfully embodied by the work and recognized there. Think of it! How frivolous it would be!